Bar Exam Final Stretch Issues

Pataasin ang iyong marka sa homework at exams ngayon gamit ang Quizwiz!

Cy Pres

"As near as possible" If a given disposition violates the rule, a court may reform it in a way that most closely matches grantor's intent while complying with the RAP.

void marriages

(1) bigamy (can become legit if first marriage ends) (2) consanguinity (parents & kids; bros & sis.; uncles/ant) (3) same-sex (4) under 16 (unless court order) have no legal status and do not require a legal proceeding to invalidate-ANYONE can attack a void marriage-Example: a bigamous marriage

FAMILY LAW essay topic KEY 1: COMMON LAW MARRIAGE - Usually not found to exist on the MEE - If a couple is common law married in a state that recognizes it, other states will recognize the marriage too. - COMMON LAW MARRIAGE ELEMENTS (CACH) - Recognition of marriage - End of Marriage fault vs. no fault

- Common law marriage: to establish a common law marriage, the proponent must show (mnemonic = "CACH"): capacity to enter into a marital contract, a present agreement to be married, cohabitation, and "holding out" a marital relationship. -Recognition of marriage: a marriage valid under the law of the place in which it was contracted will be valid elsewhere unless it violates a strong public policy of the state that has the most significant relationship to the spouses and the marriage. - End of marriage: Virtually all states are no-fault divorce states. Some states recognize fault-based grounds too. Annulment is also recognized for fraud and other reasons.

Transfer by Mortgagor - subject to -assumes

- subject to - original mortgagor is liable still - assumes - both are liable

Non-Hearsay (Avalibility Immaterial) (Tip: Don't make this harder than it is. If a party says something, it can be used against them without a hearsay issue.) AI - Oblivion, Apreciate Power, Elon, Capichulate

1) Any statement made by the opposing party that is offered against that party 2) Adoptive admissions (statements a person adopts through silence) 3) Agent/employee statements made by the agent/employee offered against the principal during the existence of the relationship and concerning a matter within the scope of the agency/employment 4) Statements by co-conspirators made during the course of and in furtherance of the conspiracy

Family Law There are generally 8 requirements for a valid marriage:

1) mental capacity 2) neither party can be married to someone else 3) age requirement(s) 4) unrelated parties (incest provisions) 5) knowing and voluntary consent 6) parties to different sexes 7) procedural requirements met 8) marriage solemnized

Special Made Goods:

1) specially made for buyer 2) substantial begining of work 3) cant sell in ordinary course of business

voidable marriages

1. Defect Which Could Render the Marriage Void a. Marriage is valid until declared void b. Subject to direct attack only - in action for annulment VALID UNTIL CHALLENGED and must be annulled in order to be dissolved-they can also be ratified-ONLY THE PARTIES can attack a voidable marriage- Example: a marriage contracted by a 17 year old w/o parental consent

Mirror Image Rule

A common law rule that requires that the terms of the offeree's acceptance adhere exactly to the terms of the offeror's offer for a valid contract to be formed.

A city ordinance provided that anyone who wanted to speak in a public park must have a permit to do so issued by the city. The ordinance granted the mayor the power to issue or deny such permits based on the mayor's judgment of whether the speech would be "in the public interest." The mayor has never denied a permit to anybody desiring to speak on a political topic. A city resident who was unhappy with the city government went to a public park in the city square. There, the resident made a 10-minute speech accusing the mayor and the city council of gross incompetence and urging voters to "throw the rascals out" at the next election. The city resident had not applied for a permit. After the resident completed his oration, the police arrested him and charged him with violating the permit ordinance. Would a conviction of the resident be constitutional?

A conviction of the resident would not be constitutional because the ordinance is void on its face. Although a municipality can place reasonable time, place, and manner restrictions on certain aspects of speech, it may not adopt a regulation that gives officials broad discretion over speech issues. If a statute gives licensing officials unbridled discretion, it is void on its face, and speakers need not even apply for a permit. They may exercise their First Amendment rights even if they could have been denied a permit under a valid law, and they may not be punished for violating the licensing statute. Here, the law allows the mayor to grant or deny permits based on his assessment of public interest. This is too much discretion to be valid. Therefore, the ordinance is void.

Nature of the Corporation

A corporation is a separate legal entity distinct from its shareholders. It may exercise, through its agents, the same rights and privileges as a natural person. The principle advantages of the corporate form are: • Limited liability of shareholders for corporate obligations; • Centralized management; • Continuity of existence; • Ease of transferring ownership; and • Access to capital through the sale of shares.

Requirements to the State

A corporation qualified to do business in the state must file an annual report with the Department of State. A corporation must maintain a registered office and an agent in the state.

Priority essay: Confessions - Due Process 14th Amendment/6th Amendment If the essay tests confessions, be able to distinguish between the Fourteenth Amendment, Sixth Amendment, and Fifth Amendment, as well as when the rights under those Amendments attach. Know that anything other than an unequivocal request for counsel will not be sufficient to invoke one's Fifth Amendment Miranda rights. Also note that officers are virtually never required to tell the defendant that a lawyer is trying to contact him. Sixth Amendment—right to counsel

A defendant shall have the assistance of counsel for his defense. There are ten (10) critical states of a trial where a defendant has the right to counsel. • Custodial interrogation - after arrest or if asserted; • Accusatory stage; • Post charge line-up; • Post indictment interrogation; • Preliminary hearings; • Misdemeanor trials with a sentence of imprisonment; • Felony trials; • Guilty pleas; • Sentencing; • Appeals as a matter of right. Fourteenth Amendment—voluntariness under the Due Process Clause: The standard for excluding a confession under the Due Process Clause is (1) whether the police subjected the suspect to coercive conduct and (2) whether the conduct was sufficient to overcome the will of the suspect. Neither the confession nor fruits of the confession can be admitted if the Fourteenth Amendment is violated. The Sixth Amendment, as applied to the states through the Fourteenth Amendment, provides that the accused has the right "to have Assistance of Counsel for his defense." § Itattacheswhenjudicialproceedingshavebegun—thatis,whentheaccusedisformally charged via indictment, arraignment, preliminary hearing, etc. It does not attach upon arrest. It applies to all "critical stages" of the prosecution after formal charges are filed. § Once it attaches, any attempts to deliberately elicit an incriminating statement about the offense that the defendant was charged with, in the absence of counsel or a knowing, intelligent, and voluntary waiver, violates the Sixth Amendment.

Lay Person Opinion

A lay person acting as a witness may state an opinion or to give inferences when testifying; first hand knowledge is required. Lay opinions are allowed to give: • Opinions or inferences that are rationally based on perceptions; and that are helpful to the jury; and not based on specialized knowledge. • Testimony to prove character when evidence of character is admissible.

Limited Liability Companies

A limited liability company (LLC) is a statutory form of a business organization designed to give investors both limited liability as in a corporation and federal (but not state) income taxation as a partnership. A limited liability company has most of the statutory powers accorded to corporations, including the capacity to sue or be sued as an entity, to hold property and to adopt an operating agreement to govern its internal affairs. ❖ Bar Professors' Tip: Members of an LLC cannot maintain a direct action against the manager of the LLC. Members of a manager-managed LLC do not have the right to maintain a direct action against the manager of the LLC when the alleged misconduct caused harm only to the LLC. The overwhelming rule is that a claim for an injury to a corporate entity against a manager or officer must be brought derivatively. ❖ Bar Professors' Tip: Generally, members of a LLC are immune from personal liabilities. However, managing members are not immune in carrying out their duties as members. Thus, if managing members breach a fiduciary duty to the other members, they can be personally liable. ❖ Bar Professors' Tip: An LLC can be dissolved when its members no longer agree and it cannot longer achieve its purpose. An LLC can also be dissolved when one party acts to the unfair detriment of the other party.

Negligence per se: discuss this doctrine when you see a statute that sets the standard of care on a Torts question.

A plaintiff can sue under a theory of negligence per se when the plaintiff can show three elements: (1) the defendant violated a statute without excuse, (2) the plaintiff was in the class of people that the statute was trying to protect, and (3) the plaintiff received the injury that the statute was trying to prevent. If a plaintiff can establish the above elements, he has offered conclusive proof of duty and breach. (He must still prove cause and harm.)

Comparative Negligence

A rule in tort law, used in the majority of states, that reduces the plaintiff's recovery in proportion to the plaintiff's degree of fault, rather than barring recovery completely. pure - reduced by p's fault (DEFAULT) partial - more at fault - p cannot recover Contributory - p cannot recover if he was at any fauult. or had last clear chance (DONT USE UNLESS TOLD TO)

Corporate Powers and Liabilities

A. Corporate Powers There are broad statutory powers conferred upon all corporations to allow the corporation to carry out its purposes. Corporate power may be limited or expanded by the Articles of Incorporation. B. Corporate Liabilities Corporations are liable for their contracts and for torts committed by their agents. Punitive damages can be assessed. C. Ultra Vires Doctrine The ultra vires doctrine concerns those acts that go beyond the power of the corporation conferred by law or by its charter . At common law, ultra vires could be asserted as a defense, e.g., to avoid a contract. This defense has generally been abolished except: • A shareholder derivative suit may be brought against the corporation to enjoin performance of a specific ultra vires executory contract; and • The corporation may bring a lawsuit against officers or directors to recover damages for past ultra vires acts that damaged the corporation.

Family Law Key Principle #4: Be aware of the three factors that a court will look at when considering to grant spousal support ("alimony"). Also, be aware of when a court may modify or terminate alimony.

Alimony: Alimony can be permanent, temporary, or granted in a lump sum. Almost all states require the trial court to consider the parties' financial resources and needs, marital contributions, and marital duration. Some states also look at spousal misconduct, one spouse's support for the other's education or training, etc. The trial court has substantial discretion in choosing to award alimony. Modification and termination of alimony: Alimony awards may be modified if a court finds there has been a substantial change in circumstances. This must be an unanticipated change. Alimony usually terminates if a spouse dies or gets remarried. In some states, cohabitation will reduce or terminate alimony.

Key Principle #5: Know that the difference between anticipatory repudiation and prospective inability to perform is truly one of degree.

Anticipatory repudiation: This occurs when there is an unequivocal manifestation by one party to the other that the party cannot or will not perform its obligations under the contract (a mere expression of doubt is not enough) and this statement is made before the repudiating party's performance is due. The other party may wait for a reasonable time for performance or resort to any remedy for breach of contract. Prospective inability to perform: This occurs when a party has reasonable grounds for insecurity that the other party is unable or unwilling to perform. This is merely doubt, it does not rise to the level of an anticipatory repudiation. Under the UCC, the party may then, in writing, demand adequate assurance of performance, and until she receives such assurance, may suspend her performance. If such assurance is not given within a reasonable time, not exceeding 30 days, the other party may treat it as a repudiation. Retracting a repudiation: the party who has repudiated can retract his repudiation unless the other party cancelled the contract, materially changed his position in reliance on the repudiation, or indicated that she considers the repudiation to be final.

counter offer

Any response to an offer (other than outright rejection) which presents alternative terms for a contract.

Key Principle #1: State the following key introduction found in the first bullet point below if Article 2 is at issue. Article 2 of the Uniform Commercial Code (UCC) is tested about half of the time that there is a Contracts and Sales question on the MEE.

Article 2 of the Uniform Commercial Code (UCC) applies to transactions in goods. Goods are "things moveable" at the time of identification to the contract. A contract under Article 2 may be made in "any manner sufficient to show agreement, including conduct by both parties which recognizes the existence of such a contract."

Articles and Defects to formation

Articles of Incorporation The Articles of Incorporation constitutes the agreement among the incorporators regarding the details of the corporate organization. The Articles must state: • Corporation name; • Number of shares and the distinguishing characteristics of each class or series; • Whether shareholders have preemptive rights; • Address of the registered office; • Name of the registered agent; • Names and addresses of the incorporators; • Address of the principal. The Articles may include: • Number of directors; • Par value of stock; • Imposition of personal liability on shareholders; • Initial purposes; and • Any other provision. The corporation's existence begins upon filing. Common law concepts of de jure corporation, de facto corporation, or corporation by estoppel have limited application under some state laws, where the filing of the Articles is deemed conclusive evidence of a valid incorporation. • De Jure Corporation A de jure corporation is a corporation formed under substantial statutory compliance and, therefore, its corporation status cannot be challenged. • De Facto Corporation A de facto corporation is when a corporation may exist even if there is a substantial defect in formation, provided there is a good faith effort to incorporate, colorable compliance with law and actual use of corporation status. • Corporation by Estoppel Corporation by Estoppel is an equitable doctrine that may be applied when persons have dealt with a defectively formed corporation as if it were a legal corporation. • Disregard of Corporate Entity/Piercing the Corporate Veil A court may disregard the corporate entity or "pierce the corporate veil" in certain situations. Piercing the corporate veil is used to describe the action of a court to hold corporate shareholders and LLC owners personally liable for the debts and liabilities of a corporation. Inactive shareholders are generally not held liable. ❖ Bar Professors' Tip: To pierce the corporate veil of an LLC and hold the members personally liable, a claimant must demonstrate either that the LLC was a mere instrumentality of the individual defendants or that there was a unity of interest and ownership between the LLC and the individual defendants. ❖ Bar Professors' Tip: Under the mere instrumentality test, one must show that the members committed a fraud or wrong that caused the injury complained of. Under the unity of interest ownership test, one must demonstrate that there was such a unity of interest and ownership between the entity and the members, and in fact, did not have an existence independent of the other members. • Alter Ego Doctrine The court might disregard the corporate entity when the corporation appears to be the alter ego of the shareholders and used by them as a conduit for their personal affairs. Some state laws require a showing of improper conduct. • Thin Capitalization A corporation must have capital adequate to meet its reasonably foreseeable needs.

A defendant is on trial for stealing jewelry from his co-worker. The defendant claims that the co-worker sold the jewelry to him because she needed money to buy medicine for her sick mother. The defense witness is asked to testify as to the co-worker's reputation in the community. The witness testifies that the co-worker is known as a dishonest person who makes her living as a "con artist." Assuming appropriate objections by defense counsel, which of the following questions would NOT be proper on cross-examination of the witness by the prosecutor? A"Isn't it true that you're maligning the defendant's co-worker because she and your wife have been enemies since childhood? B"Isn't it true that you were charged last year with assault for striking your wife?" C"Have you heard that the defendant's co-worker teaches Sunday School classes on morality and has received an award from her church based on her outstanding moral character?" "Do you know that the defendant's co-worker teaches Sunday School classes on morality and has received an award from her church based on her outstanding moral character?"

Asking the witness about the assault charge is an improper method of impeachment. A witness may be interrogated upon cross-examination with respect to an act of misconduct only if it is probative of truthfulness. An assault is not probative of truthfulness, so it would not be proper impeachment evidence. Had the witness been convicted of the assault, the conviction would have been admissible, provided it was a felony. (A) is incorrect because it is an example of proper impeachment by showing bias. Evidence that a witness is biased tends to show that he has a motive to lie. The witness's close relationship to his wife gives rise to an inference that he would be hostile toward the co-worker if she and his wife had a long-standing personal enmity. Consequently, the question posed in (A) represents a proper method of impeaching the witness's credibility by probing into a possible bias against the co-worker. (C) and (D) are incorrect because these questions represent proper means of rebutting the evidence of the co-worker's character for dishonesty, as well as trying to impeach the witness's credibility based on lack of knowledge. Once the defendant has introduced evidence of the alleged victim's bad character for a pertinent trait, the prosecution may counter with reputation or opinion evidence of the victim's good character for the same trait. [Fed. R. Evid. 404] On cross-examination, the prosecution may inquire into relevant specific instances of conduct. [Fed. R. Evid. 405(a)] Traditionally, asking a witness if he has heard of a particular instance of conduct represents a means of testing the accuracy of the hearing and reporting of a reputation witness, who relates what he has heard. Asking a witness if he knows of a particular instance of conduct is a means of testing the basis of an opinion expressed by the witness. Here, the witness's testimony indicates both that he has heard that the co-worker has a bad reputation for honesty and that his own opinion is that she is a dishonest person. Thus, in attempting to rebut this testimony, the prosecution may test the accuracy of what the witness has heard concerning the co-worker's character by asking him if he has heard of specific instances of her teaching Sunday School and receiving a church award. Also, the prosecution may test the basis for the witness's opinion as to the co-worker's dishonesty by asking if he knows of these specific instances that are indicative of her good character.

Key Principle #2: The two intentional torts that have been tested on the MEE are battery and false imprisonment. Intentional torts are not heavily tested.

Battery: An act with intent to cause a harmful or offensive contact or imminent apprehension of that contact and a harmful or offensive contact directly or indirectly results. Note that the primary difference between an assault and a battery is the harm suffered (for an assault, the plaintiff suffers imminent apprehension) False imprisonment: an act with intent to confine or restrain a person to a bounded area, actual confinement occurs, and the plaintiff knows of confinement or is hurt by confinement. Consent: Consent is a defense to an intentional tort. Consent can be express or implied.

conflict of laws family law

BASIC QUESTION: Will one state recognize a marriage that has been entered into in another state?RULE: the law of the state where the marriage contract was entered into will determine its validity EXCEPTIONS:1) if the state being asked to recognize the marriage has a strong public policy against that type of marriage, then the marriage may not be recognized2) where there is a specific statutory prohibition in the state being asked to recognize the marriage (Example: same sex marriages)

Family law Key #2 Custody mod of custody child support mod of child support relocation parental rights

BIOC - Modifications of Custody - Substantial Change in circumstances States use numerical guidelines - income, where kids live, ages of the children, special needs, current life etc. Mod of CS - Substantial Change in circumstances Relocation of parent and child: Generally, a move sought in good faith that will serve the child's best interest will ordinarily be approved. The court will balance the impact on visitation by the noncustodial parent against the benefits of the move to both the child(ren) and the custodial parent. There are a few different views (which you should mention if applicable): some states place the burden on the relocating parent and some put it on the objecting parent when does the father have rights? Biological fathers generally have rights. However, the state may make the parent exercise his rights within a specific time (e.g., two years) Rights of a parent when another person wants to adopt a child: an involved parent who demonstrates a "full commitment to the responsibilities of parenthood" will likely be able to successfully oppose an adoption petition by another and is entitled to notice of such proceeding Third-party rights: Custody in the parent is presumed to be in the best interest of the child. To rebut this, a third party who wants custody must prove that the parent is unfit or that granting custody to the parent would be highly detrimental to the child. Thus, any third-party visitation or custody statute must give special weight to the parent's determination of the child's best interest

A construction company that was putting in a swimming pool for a homeowner left a couple of large pieces of equipment in the backyard overnight. The equipment was not owned by the construction company but was leased from an equipment company, which was responsible for its repair and maintenance. After the workers had left, a seven-year-old boy came onto the homeowner's property to play. The homeowner was aware that the boy often came onto his property to play with his dog. The boy climbed up on one of the pieces of equipment and began pushing buttons and moving levers. The engine started and the equipment began to move because the equipment company had not replaced a defective safety locking device on the ignition. The boy became frightened and jumped off, falling into the hole that had been dug that day, and was injured. The boy's parents brought suit against the homeowner and the construction company. If the construction company is held liable for the boy's injuries, may it recover anything from other tortfeasors?

Because the equipment company negligently maintained the equipment, the construction company could obtain contribution from the equipment company. When two or more tortious acts combine to proximately cause an indivisible injury to a plaintiff, each tortious actor will be jointly and severally liable for that injury. Joint and several liability permits a plaintiff to recover the entire judgment amount from any defendant. Contribution allows a defendant required to pay more than his share of damages to recover from the other jointly liable parties for the excess. In other words, contribution apportions responsibility among those who are at fault. Here, if the construction company is held liable for the boy's injuries, it will be because of its negligence in leaving unattended a piece of equipment without a working safety locking device. However, because the equipment company, which was responsible for repair and maintenance of the equipment, negligently performed such maintenance, resulting in the absence of a working safety locking device, then the equipment company's negligence would have combined with that of the construction company to proximately cause the boy's injuries. This would render the companies jointly and severally liable to the boy for the entire damage incurred. Thus, if the construction company is held liable for the injuries, it has a claim against the equipment company, as a jointly liable party, for the amount it pays in excess of its share of damages. Note that, in the usual case, the equipment company would have been included in the lawsuit. However, the fact that it was not included does not preclude the construction company from recovering contribution in a separate action. (A) is incorrect because indemnity is not available here. Indemnity involves shifting the entire loss between or among tortfeasors, and is available where: (i) there is a contractual promise to indemnify; (ii) there is a special relationship between the defendants that would allow for vicarious liability; or (iii) the defendant is a supplier in a strict products liability case who is liable to an injured customer, thus giving the supplier a right of indemnification against previous suppliers in the distribution chain. In addition, some states allow a joint tortfeasor to recover indemnification from a co-joint tortfeasor where there is a considerable difference in degree of fault. Here, there is no evidence of a contractual right to indemnity between the construction company and the equipment company, there is no relationship between them that causes the construction company to be held vicariously liable for the equipment company's negligence, and this is not a strict products liability case. Also, there is no indication of a considerable difference in degree of fault between the two companies. Therefore, none of the circumstances in which indemnity is available is present. (C) is incorrect. As to the homeowner, it is doubtful that he would be liable because he did not know (and had no reason to know) that the safety locking device on the equipment was missing. Even if a jury were to find him, as the property owner, at fault to some degree, indemnity would not be applicable to this situation, as discussed above. (D) is incorrect because, as explained above, the construction company can recover from the equipment company based on contribution rules.

Why is it being said? To prove it's truth? Declarant may be available or unavailable

Business Records Excited Utterance Present Sense Impression Present State of Mind Prior Convictions/JudgmentsPast Record Recollection Physical Condition Public Records LMFAD Ancient Document Learned Doctrine

Other Corp issues

Bylaws are the written rules for the conduct of a corporation, association, partnership or any organization. Bylaws generally provide for meetings, elections of a board of directors and officers, filling vacancies, notices, types and duties of officers, committees, assessments and other routine conduct. Bylaws provide the framework for the operation and management of the corporation. Bylaws are a contract among members, and must be formally adopted and/or amended. ❖ Bar Professors' Tip: Any bylaw provision that conflicts with the articles of incorporation is void. 6. Finance A. Capital Structure The capital structure refers to long-term investment in the corporation, including stock and long-term debt such as bonds. B. Authorized Capital Authorized capital refers to the number and kinds of shares provided for in the Articles of Incorporation, whether or not the shares are actually issued. There must be at least one class of common stock (voting or nonvoting) representing the residual ownership of the corporation. Preferred stock (voting or nonvoting) may be of several different kinds, generally with a right to be paid a fixed dividend ahead of any dividend payments to holders of common stock. C. Agreements to Buy Stock • Subscription Agreements A subscription agreement is a contract under which the subscriber agrees to purchase a certain number of shares at a specified price. At common law, subscription agreements were generally revocable until adopted by the corporation.

To find sufficient minimum contacts over a defendant for MOP, the defendant must eat at CHILIs

C- ∆ CONSENTS to jurisdiction in the Forum State--> through SAMH-the state where the corporation or individual is at HOME (corp--> Nerve Center, individual--> domicile) I - the state where a corporate defendant was INCORPORATED L- ∆ is a non-domiciliary, but the court has LONG-ARM jurisdiction (--> RIOT & 3-S) IS- ∆, non-domicilary, is served IN STATE (Gen. Jurisdiction)

A plaintiff brought an action in federal court against a state government, seeking monetary damages. The state moved to have the case dismissed for lack of jurisdiction, citing the Eleventh Amendment of the United States Constitution. Which of the following facts would support a denial of the state's motion? AThe plaintiff is a private citizen of the defendant state. BThe plaintiff is a Native American tribe. CThe plaintiff is a neighboring state. DThe plaintiff is a foreign government.

CThe plaintiff is a neighboring state.

Con Law Clauses Applied to Fed Gov

CC - congress can act if activity is comercial. effects interstate commerce. Welfare Clause - tax and spend, valid if no violation of other con law

Civil Cases Chacter Evidence

Character evidence in civil cases is usually inadmissible to prove conduct in conformity with the character. Character evidence is admissible and could be proven by reputation, opinion, or specific acts, in cases where character is an essential element of the claim or defense, e.g., defamation.

Character Evidence

Character evidence is evidence that shows that a person has a particular disposition and acted in accordance with that disposition at the time in question. Character evidence is inadmissible for the purposes of proving action in conformity therewith on a particular occasion. These are the exceptions to character evidence inadmissibility:

Key Principle #6: Be familiar with the Statute of Frauds.

Common law: State: "A contract within the Statute of Frauds satisfies that statute and is enforceable if it is evidenced by a writing signed by 'the party to be charged,' which (1) reasonably identifies the subject matter of the contract, (2) is sufficient to indicate that a contract has been made, and (3) states with reasonable certainty the essential terms of the contract." Contracts that are within the statute include contracts for the sale of an interest in land, contracts that cannot be performed in a year, contracts made in consideration of marriage, promises by an executor to pay a debt of an estate out of his personal funds, and suretyship contracts UCC: A contract for the sale of goods for a price of $500 or more is not enforceable unless there is a writing signed by the party against whom enforcement is sought that is sufficient to indicate that a contract for the sale of goods has been made between the parties. The writing need not contain all terms of the contract, but it is not enforceable beyond the quantity of the goods shown. Exceptions to be aware of are the confirmatory memo exception, the part performance exception, the specially manufactured goods exception, and the judicial admissions exception.

Validity of Statues question: is it Fed or State? If it's Fed...

Congress can only act pursuant to enumerated powers. a valid statute must: - be rationally related to enumerated power or be nec. and propert to effecuate an enumerated power.

Fixtures - DAMN

Damage from removal Adaptation of item Manner in which attached Nature of item

What Makes title unmarketable? D E V A

Defect in chain of title Encumbrance (mortgage or easement not mentioned in the contract) Violation of zoning ordinance Adverse posession - title aquired by

Key Principle #3: Be familiar with how a court will divide property in a divorce proceeding. Divsion by Court Marital v. Separate prop. (Majority) (Big) PreNup

Division by court: in a majority of states, marital property is divided at divorce but separate property remains the property of the owning spouse. Martial vs. separate property (majority approach): Marital property is property acquired during the marriage. Separate property includes (mnemonic = "BIG") property acquired before the marriage, an inheritance, or a gift to one party. Most states do not count professional degrees earned during the marriage as marital property. Division based on premarital agreement: A court will enforce a premarital agreement so long as it is voluntarily made, substantively fair, and if full disclosure of assets and obligations was made. A court will not, however, enforce a premarital agreement regarding child custody or support if it is not in the best interest of the child.

Con Law Clauses Applied to states

Dormant CC -limit Contracts Clause - Limit P&I - states cant deny P&I of national citizenship (weak answer choice) P&I Art 4 - No discrim against out of staters unless substantial purpose

Key Principle #7: Be familiar with duress, misrepresentation, and other contractual defenses.

Duress: a contract is voidable when it is established that a party's manifestation of assent is induced by an improper threat that leaves the party no reasonable alternative. Breach of Express Warranty: Under Article 2, affirmations of fact relating to the goods that are part of the basis of the bargain create express warranties that the goods will conform to those affirmations and descriptions. This warranty is breached if they do not conform.

Expert Testimony on Ultimate Issues

Expert testimony is permissible even if it addresses the ultimate issue in the case. In a criminal case, the expert cannot render an opinion as to the mental state of a defendant.

Key Principle #3: Vicarious liability is heavily tested. Understand when an employer is liable and when the employer is able to seek indemnification for damages paid to the plaintiff.

Employer/employee: Employers are vicariously liable for torts of their employees if the torts are committed within the scope of employment. (This is called "respondeat superior.") Intentional torts are usually outside the scope of employment unless they were done for the purpose of serving the employer or if they were foreseeable. Direct liability: the employer can be directly liable for his own negligence if, for example, he fails to supervise employees or otherwise acts negligently in hiring, firing, or entrustment Indemnification: Indemnification is full reimbursement for damages paid to the plaintiff. This is when one defendant can seek 100% of the damages from the other defendant. This usually occurs when the paying defendant was not at fault in causing the plaintiff's injuries, and the non-paying defendant was at fault. For example, an employer who is liable based solely on the principle of respondeat superior may seek indemnification from the employee who was negligent.

Liability Insurance ❖ Bar Professors' Tip: If you see a liability insurance question, look for the underlying reason for admissibility. It is probably an ownership or control issue and thus, admissible.

Evidence of liability insurance or lack thereof is inadmissible to prove the defendant was negligent or that the defendant can pay a judgment. It is admissible for impeachment purposes, to prove an agency relationship between a witness and a defendant, or to show ownership, control or bias of a witness.

Key Principle #7: Note that final judgments are generally appealable and non-final judgments are generally not appealable. However, the MEE tends to test the exceptions!

Exceptions include when there are final orders in cases involving multiple claims and multiple parties and some are still pending—the other parties may be able to appeal their orders; orders involving injunctions, garnishments, and other temporary remedies; interlocutory orders by leave; and orders constituting a final judgment on collateral matters.

Was breech minor or material? EH ANAL

Extent of breech Hardship Amount of Benefit Negligence of breaching party Adequacy of compensation Likelyhood of breeching party's consideraiton

Why is it being said? To prove it's truth? Hearsay Exceptions - Unavailable Witness (i.e., invokes a privilege, is absent from the jurisdiction, is ill or dead, lacks memory, or refuses to testify) FaT DD HIC doesn't need to be available PRISM = Unavailable

FaT - Former Testimony DD - Dying Declaration H - personal or family History I - statement against Interset C - statements offered against party that wrongfully Caused D's unavailability P - Privilege R - Refusal I - Illness or Death S - Subpoena M - no Memory of subject matter

CIV PRO Key Principle #2: Subject matter jurisdiction (SMJ) is the power of a court to hear a certain type of case. Federal courts are courts of limited jurisdiction—that is, they can only hear certain types of cases. When SMJ is tested, the issues have been predictable. There are three categories of SMJ that are tested:

First category of federal jurisdiction—federal question jurisdiction: The federal question must appear on the plaintiff's well-plead complaint. It cannot appear in the answer; further, the plaintiff cannot merely anticipate a federal defense in its complaint. Second category of federal jurisdiction—diversity jurisdiction: cases may be brought under diversity jurisdiction only if two requirements are met: (1) there must be complete diversity of citizenship between the plaintiffs and defendants, and (2) the amount in controversy must be over $75,000.00. Note that "complete diversity" is not required for class actions; rather, minimal diversity suffices. Many essays focus on where a person is domiciled. Remember, a person is domiciled "where it is her permanent home, a place where the person intends to remain indefinitely, and the place to which the person intends to return when temporarily absent." Look to see where the person is domiciled at the time the lawsuit is filed. Third category of federal jurisdiction—supplemental jurisdiction: This is an issue when there is a jurisdictional basis for one claim but not the other (e.g., a plaintiff brings a "federal question" claim and tacks on a related state claim). Remember that a plaintiff cannot use supplemental jurisdiction to add a claim against a non-diverse party if the sole basis for SMJ is diversity. A note on removal under 1441: Defendants (but not plaintiffs) may remove an action from state court to the federal court that geographically embraces it if the plaintiff could have initially brought the case in federal court. Generally, if the plaintiff could not have brought the case in federal court, then the defendant cannot remove it either.

Key Principle #8: Be familiar with general damage principles.

General damages for breach of contract: Start your essay as follows: "The normal measure of damages for breach of contract is expectation damages, which aim to give the non-breaching party the benefit of his bargain." Expectation damages must be foreseeable and proved with reasonable certainty. This is similar to the UCC rule, which puts the aggrieved party "in as good a position as if the other party had fully performed. Punitive damages: note that punitive damages are not generally recoverable as an element of damages in a breach of contract action "unless the conduct constituting the breach is also a tort for which punitive damages can be recovered. Restitution (also called unjust enrichment and quantum meruit): a party may be able to recover restitution for any benefit conferred by way of part performance in excess of the loss that he caused by his own breach. Mitigation: a party must mitigate damages when possible.

Hearsay Essay DIA -Define Hearsay -Important - Analyze exceptions ❖ Bar Professors' Tip: Watch for hearsay within hearsay. When you see a statement made by a witness in the official report, there must be a way to have that statement qualify as a hearsay exception or it will be excluded despite the report qualifying as a hearsay exception.

Hearsay is an out of court statement offered to prove the truth of the matter contained in that statement. A "statement" includes oral or written assertions, and non-verbal conduct intended by its maker to be an assertion. If the statement is hearsay and there are no exceptions, the statement is excluded. - Important bc it is not admissible unless there is an exception

Execution of a Deed

Ids parties has words of grant signed by the seller describes the land Delivery means intent to pass title presently - presued to have occured is the deed is in the grantee's posesssion or if it is recorded. Marketable title needs to be given at closing - not before

Priority Essay Topic Crim Law USE ABOVE 4A ANALYSIS Plain View Doctrine/Auto Exception

If there is probable cause to search a lawfully stopped automobile, the police may search any part of the automobile, including containers that may conceal the object of the search. Even if the police have made a lawful custodial arrest of the occupant of the automobile, they cannot use the search incident to arrest to search the passenger compartment or the contents of any containers found unless they have a reasonable fear that the driver might gain access to the compartment or they have probable cause to suspect that evidence of a crime or contraband will be found. Plain View Pursuant to the plain view doctrine, the police are not in violation of the 4th Amendment search when they view objects that are in plain view of the officer who has the right to be in the position to have that view. Note any specific language of the right to be in the position to view the evidence. Plain view exception: if officers are lawfully in a position from which they view an object, if its incriminating character is immediately apparent, and if the officers have a lawful right of access to it, they may seize it without a warrant. ❖ Bar Professors' Tip: Trespass on neighbor's property to get the plain view: If the police gain the view of defendant's activities by trespassing on the property of the defendant's neighbor, the fact that the police violated the neighbor's legal rights will not prevent the plain view doctrine from applying against the defendant.

Key principle #1: Negligence is by far the most highly tested topic on Torts MEE questions. Be familiar with the general standard of care (to act as a reasonable person would) and when the general standard changes.

Introduction: start your essay as follows: "In any negligence action, a plaintiff must show that the defendant owed the plaintiff a duty to conform his conduct to a standard necessary to avoid an unreasonable risk of harm to others, that the defendant's conduct fell below the applicable standard of care, and that the defendant's conduct was both the cause in fact and the proximate cause of the plaintiff's injuries." - then to into duties - then if applicable trespass duties or duties of LO

Key Principle #8: Be aware of issue and claim preclusion.

Issue preclusion: issues that were actually litigated and decided and essential to the judgment in a previous case cannot be litigated again. Claim preclusion: a claim that has been litigated to a final judgment on the merits cannot be re- litigated by the parties (or their privies).

Key Principle #3: Be familiar with the variety of trusts that can be created. The following have been tested:

LOOK AT SHEET

To obtain valid Procedural Jurisdiction, 14th amendment due process clause requires the defendant have MOP:

M- some MINIMUM contact with the forum state O- an OPPORTUNITY to be heard in the suitP- been PROPERLY served with process

Marriage Communication or Spousal Immunity

Marriage Communication This privilege allows a witness spouse to refuse to testify about confidential communications made during the marriage. Either spouse has the privilege. Even after a divorce, a spouse cannot testify as to matters learned during the marriage. ❖ Bar Professors' Tip: Look for an overheard conversation to break the confidentiality. Spousal Immunity This privilege is only used in criminal cases. No confidential communication is needed. After divorce, the spouse could testify as to matters learned before and during marriage. In federal criminal courts, the witness spouse has the privilege.

Criminal or Civil Cases

Mimic PO Past conduct is admissible against a defendant to establish an element of a present crime; past conduct is admissible to prove the defendant's: motive, intent, lack of mistake, identity, common plan or scheme, preparation, and opportunity. The evidence may not be offered in the prosecution's case-in-chief but may be offered in rebuttal if the specific trait to be proven is at issue. • Sexual Misconduct In a case alleging sexual assault or child molestation, prior specific sexual misconduct of the defendant is admissible as part of the prosecution's case in chief or any relevant purpose including the defendant's propensity to commit the crime. D. Proving Character Where character evidence is admissible, proof may be made by: • Reputation or Opinion Evidence Testimony in the form of reputation or in the form of an opinion. • Specific Instances of Conduct Specific instances of conduct may also be used as proof when a person's character or character trait is an essential element of a charge, claim or defense. 3. Authentication of Writings A. Authentication After determining relevancy, a writing must be authenticated before it is admitted into evidence. A foundation is laid to allow a finding of what the writing purports to be. Ancient Documents Rule This rule allows a document to be authenticated by evidence that was prepared before January 1, 1998, and whose authenticity is established. The common law rule is thirty years. ❖ Bar Professors' Tip: The Supreme Court adopted the new Ancient Document Rule in 2017. Prior to the new rule adoption, the document to be authenticated had to be at least 20 years. C. Self-Authenticating Documents These are documents that are self-proving and require no authentication, e.g., official publications, certified copies of public records, documents accompanied by a certificate or authentication, commercial paper, material in newspapers or magazines, trade inscriptions on business materials. The Supreme Court, in 2017, adopted the following certified electronic records • Certified Records Generated by an Electronic Process or System: A record generated by an electronic process or system that produces an accurate result, as shown by a certification of a qualified person that complies with the certification requirements of Rule 902(11) or (12). The proponent must also meet the notice requirements of Rule 902(11). • Certified Data Copied from an Electronic Device, Storage Medium, or File: Data copied from an electronic device, storage medium, or file, if authenticated by a process of digital identification, as shown by a certification of a qualified person that complies with the certification requirements of Rule 902(11) or (12). The proponent also must meet the notice requirements of Rule 902(11). D. The Best Evidence Rule Sometimes called the Original Document Rule. To prove the content of a writing, recording (audio or video), or photo. the proponent must offer the original, subject to some exceptions. The best evidence rule applies to legally operative documents, writings offered to prove an event, or when the testimony is reliant on the writing and not on personal knowledge. If there is no original, then copies or oral testimony is admissible after it is proven the original is unavailable. Duplicates are admissible as originals in federal court unless the authenticity is challenged. The best evidence rule does not apply to prove the writing existed or that a statement was made; or when facts to be proven exist independently of any legally operative writing or when the evidence is collateral to the litigated issue.

High Priority Essay 5th Amendment - Miranda Warnings - Self Incrimination 5A State: "Law enforcement officers are required to read Miranda warnings to a suspect when the suspect is subjected to an in-custody interrogation." - Custody -Waiver -Invocation -interrogation meaning

No person shall be compelled, in a criminal case, to be a witness against himself (this right must be asserted to be effective). The privilege against self-incrimination protects against admission of testimonial evidence. Keep in mind when Miranda kicks in. No statement made by a defendant will be admitted into evidence unless, prior to custodial interrogation, the defendant is given certain warnings. Custody: a suspect is in custody if there is a formal arrest or a restraint on freedom of movement of the degree associated with a formal arrest. A valid Miranda waiver: The suspect must make a "knowing, intelligent, and voluntary" waiver. This is a low bar. Prosecution has burden A valid Miranda invocation :, the suspect must be explicit, unambiguous, and unequivocal in making the request (e.g., "I think I need a lawyer" is not enough). Interrogation: this includes questioning initiated by law enforcement officers or any words or actions that the police should know are reasonably likely to lead to an incriminating response from the suspect. Other tips ❖ Bar Professors' Tip: Trickery and deceit does not render a confession inadmissible. Far from making the police a fiduciary of the suspect, the law permits the police to pressure and cajole, conceal material facts and actively mislead. ❖ Bar Professors' Tip: Courts consider the totality of the circumstances, such as age, level of education, and familiarity with the criminal justice system. ❖ Bar Professors' Tip: If an in-custody suspect, in response to Miranda warnings, invokes his right to counsel, law enforcement may re-initiate contact with the suspect if the suspect experiences a break in police custody of at least 14 days. Miranda warnings should be re-advised. ❖ Bar Professors' Tip: For a person to validly invoke the right to counsel or the right to silence, the invocation must be clear and explicit. ❖ Bar Professors' Tip: The Due Process Clause requires that a person's waiver of Miranda rights be made both knowingly and voluntarily. The court will consider facts like how long the suspect was held in a cell, or the length and types of questioning made by the police to the suspect in order to determine if there was undue pressure put on the suspect by the police. The person's ability to comprehend the charges as well as his capacity to understand the waiver of rights will also be taken into account. ❖ Bar Professors' Tip: Also look for the Sixth Amendment right to counsel.

A defendant was on trial for burglary, and he took the stand in his own defense. On direct examination, the defendant vigorously denied having committed the burglary. Also on direct examination, the defendant stated that his last regular employment was as a bookkeeper for a corporation. On cross-examination, the prosecutor asked the defendant if he had embezzled funds from the corporation. The defendant denied that he had embezzled from the corporation or from anyone else. The prosecutor then wanted to call a police officer to the stand to testify that when she arrested the defendant for embezzlement, the defendant admitted to the officer that he had embezzled money from the corporation. Assuming that the defendant has not yet been tried on the embezzlement charges, may the prosecutor call the officer to the stand?

No, because the evidence would be extrinsic. The officer may not testify about the embezzlement because it constitutes impeachment by extrinsic evidence of a specific instance of misconduct. A specific act of misconduct offered to attack the witness's character for truthfulness can be elicited only on cross-examination. If the witness denies the act, the cross-examiner cannot refute the answer by calling other witnesses or producing other evidence. Because the alleged embezzlement is admissible, if at all, only as impeachment evidence, when the defendant denied it the prosecutor could not call the officer to testify

Congress passed a bill to relocate a special forces training center. The bill included funds to facilitate the move and mandated that the funds be so spent. The bill was signed into law by the President. Thereafter, the senators from the state in which the training center is currently located informed the President that they would withdraw their support for an important bill he favored if the move takes place.Does the President have power to decline to spend the funds appropriated for the move?

No; The President has no power to decline to spend the funds appropriated for the move because they were specifically appropriated for the relocation of the training center. The President has no power to decline to spend funds specifically appropriated by Congress when Congress has expressly mandated that they be spent, regardless of Congress's reason for making the appropriation

The defendant discovered that his friend had hit and killed a pedestrian while driving that afternoon, and that he had fled from the scene of the crime before the police arrived. To keep his friend out of trouble, the defendant fixed all the dents in the car caused by the collision and had the vehicle painted a different color. The friend, distraught about hitting and killing someone, eventually turned himself in and told the police what he had done and what the defendant had done for him. The defendant was charged as an accomplice to vehicular manslaughter in a state that follows the modern trend regarding accomplice liability. How should the defendant be found?

Not guilty, because he only helped his friend after the crime was already committed. HE IS AN ACCESORRY The defendant should be found not guilty as an accomplice. Under modern statutes, parties to a crime are divided into three different categories. Principals are those who, with the requisite mental state, actually engage in the act or omission that causes the criminal result. An accomplice is one who, with the intent that the crime be committed, aids, counsels, or encourages the principal before or during the commission of the offense. Under modern statutes, accomplices are generally treated as principals. A third category is "an accessory after the fact." An accessory after the fact is one who receives, relieves, comforts, or assists another knowing that he has committed a felony, in order to help the felon escape arrest, trial, or conviction. Unlike an accomplice, an accessory after the fact has committed a separate crime with a punishment unrelated to the felony committed. In the instant case, it is clear that the defendant aided his friend in avoiding capture, that he provided no aid to the substantive offense, and that he did not intend the substantive offense to occur.

Key Principle #2: Contract formation is heavily tested. Know that offer, acceptance, and consideration are needed to form a contract, and be very familiar with each of these three parts of a contract.

Offer: A person makes an offer when the person communicates to another a statement of "willingness to enter into a bargain" so that the other understands that "his assent to the bargain is invited and will conclude it." The terms of an offer need to be reasonably certain (e.g., as to parties, subject matter, price, etc.). Compare—counteroffer:undercommonlaw,astatementisacounteroffer,ratherthan an acceptance, when the terms of the initial offer are changed—e.g., when a condition is added to the purported acceptance (Mirror Image Rule) Acceptance: An acceptance is a manifestation of assent to the terms of an offer made in a manner invited by the offer. It is effective upon dispatch (the mailbox rule). Consideration: Consideration is a "legal detriment" or "bargained-for-exchange." A promise to make a gift does not constitute consideration. UCC: only good faith needed for modification Promissory estoppel is a substitute for consideration. If there is (1) a promise, (2) reliance that is foreseeable and justifiable, and (3) enforcement is necessary to avoid injustice, the promise will be enforced.

Offers to Pay Medical or Similar Expenses

Offers to pay medical or similar expenses are inadmissible to prove a defendant's liability. Admissions coupled with offers to pay may be severed and may be admissible.

Key Principle #3: Note the difference between performance obligations under the common law and Article 2.

Performance obligations: Under common law, a party must "substantially perform" its contractual obligations in order to demand performance (usually payment) from the other party. Courts will look at several factors to determine whether performance was substantial. Note that this is different from UCC Article 2, which requires perfect tender for "one shot" deals. Exception: divisible contracts: A contract is divisible so long as (1)it is apportionable, and (2) the parties would have contracted for each part separately. A party that performs one or more parts of the contract may collect payment for those parts even if he does not substantially complete performance of his duties.

CIV PRO KEY 1 Key Principle #1: Personal jurisdiction is tested approximately once every five years on the MEE. There is a trend toward testing specific jurisdiction. Personal jurisdiction has to do with what is fair to the defendant.

Personal jurisdiction can be general (obtained by consent, presence, or domicile) or specific. If the MEE fact pattern discusses a case that takes place in a federal court, start your essay as follows: "Federal district courts may exercise personal jurisdiction to the same extent as the courts of general jurisdiction of the state in which the district court sits." If the issue is specific jurisdiction, state: "State courts of general jurisdiction may exercise personal jurisdiction over nonresident defendants to the extent authorized by both the state's long-arm statute and the due process clause of the Fourteenth Amendment of the U.S. Constitution." Then state: "The Due Process Clause of the Fourteenth Amendment permits states to assert personal jurisdiction over nonresident defendants who have established minimum contacts with the state such that the exercise of personal jurisdiction would not offend traditional notions of fair play and substantial justice." Look for "purposeful availment" of the benefits and protections of the state. Then, examine the quality of the contacts with the state.

Key Principle #2: Under the Uniform Trust Code, the "default rule" is that a trust is revocable. An irrevocable trust can still be terminated or modified in some circumstances.

Presumption of revocability: under the Uniform Trust Code, an inter vivos trust is revocable unless the instrument expressly states otherwise. Termination by settlor: a settlor may terminate the trust if all beneficiaries are in existence and all agree to the termination. Termination by beneficiaries after settlor dies: generally, even an irrevocable trust can be terminated if both the income beneficiaries and the remainder men unanimously consent and if there is no material purpose of the trust yet to be performed.

Impeachment: EE Allowed for: CaCa, Doody, PiS, Boobies, DIC

Prior Inconsistant Statements - After proper foundation is laid. Bias - After being asked about facts that show bias - Extrinsic evidence Crimincal Conviction (the conviction) is permitted. Note: if more than 10 years have passed since the witness's conviction or release from confinement for it, whichever is later, evidence of the conviction is generally not admissible. Sensory Deficency Contridiction - If W doesn't admit mistake then yet.

Methods of Impeachment

Prior Inconsistent Statements Statements made by a party at some other time that conflict or are inconsistent with a material part of his testimony can be used to impeach the witness. Bias A party can show that a witness has an interest in the outcome of the proceedings. The interest can be personal, financial or penal. First, the witness must be asked on cross-examination about the facts that show bias. If the witness admits to it, then it is within the judge's discretion to allow extrinsic evidence to prove the bias.

Consideration - legal detrient or bargined for exchange a gift is not consideration substitutes for consid:

Promissory estopple 1) promise 2) foreseeable reliance 3) enforecement nes. to prevent injustice.

Key Principle #5: Know the difference between general and special powers of appointment.

Purpose: a person writing a will or trust can give her beneficiaries a power of appointment, which enables the beneficiary to designate who will receive specific property. General power of appointment: the class of people that the beneficiary can exercise the power of appointment in favor of is unlimited (she can use it for herself, her creditors, or someone else). § Majority view: In most states, a general residuary clause in a will ("I give all of my estate . . . ") does not exercise a power of appointment. However, if the general residuary clause is coupled with a "blanket exercise" clause (e.g., "including all property over which I have a power of appointment"), any power of appointment held by the donee is exercised, unless the donor of the power specifically requires reference to it. § Minority view: a general testamentary power of appointment can be exercised by general language in the beneficiary's will (such as the residuary clause) even if it makes no reference to the power in the instrument (e.g., "Everything to my husband"—the husband will get it), unless the creating instrument of the power made an express gift in default or the instrument stated that the power needed to be specifically mentioned. Special (or limited) power of appointment: The class of people that the beneficiary can exercise the power in favor of is limited. A special testamentary power needs to be specifically exercised. The Uniform Probate Code (UPC) adopts a "substantial compliance" rule which says that if it could be shown that the powerholder intended to exercise a power, a blanket-exercise clause may be sufficient.

WITHVN

Real covenant --> BURDEN to run with the land Writing Intent Touch and concern Horizonal privity Vertical privity Notice

WITV

Real covenant BENEFIT to run with the land Writing Intent Touch and concern Vertical privity

Key Principle #4: Note the difference between a buyer's ability to reject goods versus his revocation of acceptance of goods.

Rejection of goods: A buyer can generally reject goods for any reason under the perfect tender rule. (There are exceptions to this—e.g., installment contracts.) Revocation of acceptance of goods: If a buyer accepts the goods, he can no longer reject them. However, in certain circumstances a buyer can "revoke his acceptance" of the goods. The buyer can do this when: (1) the nonconformity substantially impairs the value to him; (2) he accepted the goods because he had a reasonable belief that the nonconformity would be cured (and it was not), or he did not discover the nonconformity because the nonconformity was difficult to discover, or because of the seller's assurances; (3) he revokes within a reasonable time after he discovers or should have discovered the nonconformity; and (4) he revokes before any substantial change in condition of the goods which is not caused by their own defect. A buyer who revokes acceptance of goods may recover the purchase price that has been paid.

Formation of a corporation

Requirements Incorporators may be natural persons, corporations, partnerships, or associations. ❖ Bar Professors' Tip: To form a corporation, there must be an incorporator who executed the articles of incorporation and delivers them to the Secretary of State. The articles of incorporation must state the corporation's name, the incorporators name and address, the registered agents name and address, and authorized shares. When parties attempt to create a corporation but, unbeknownst to them, the corporation fails to come into existence, the parties will be held personally liable because what ended up forming is a general partnership. This is the default business form if an attempted corporation fails. However, if the parties were unaware of the failure to form the corporation, they can avoid personally liability by either proving that they formed a defacto corporation or there is a corporation by estoppel. A defacto corporation is proved when: (1) there is a relevant corporation statute; (2) the parties made a good faith effort to comply with the statute; and (3) the parties were acting as a corporation and exercising corporate privileges. A court will form a defacto corporation only when the parties were unaware of the mistake. Corporation by estoppel is when parties thought they formed a valid corporation and were conducting business as if they were one. They will be estopped from denying the business formed and third parties will be estopped from denying that they are a corporation.

Key Principle #6: Be familiar with the different approaches to giving gifts to a class.

Rule of convenience: when a gift is made to a group, such as "my children", the class "closes" when at least one member is entitled to distribution. (July 2012, Feb 2011, Feb 2010) UPC approach: The UPC states that when a class gift is made, each living beneficiary will take their share and the deceased beneficiary's share will pass to their surviving descendants. (If there are no surviving descendants then the gift will fail.) Note: this applies even if the beneficiary is not related to the settlor (and thus differs from most antilapse statutes). (July 2012, Feb 2011, Feb 2010, July 2008, July 2004, July 2002) Common law approach: Under the common law, if the gift or remainder to a deceased beneficiary has already vested and there is no applicable statute then it will go to whomever the instrument says it should go to or whomever the deceased person has specified in their will or through intestacy. (This also applies to gifts that are not made to classes.) (July 2018, July 2004, July 2002) If you see a gift to a class in a Wills question: if a testator gives a gift to a group of unrelated individuals and one predeceased him, the deceased would not take, and neither would his descendants, unless the antilapse statute saved the gift.

Remedies (UCC)

Seller breaches and buyer has goods: buyer gets the value of the goods as contracted for minus the value of the goods as delivered plus incidental and consequential damages. Seller breaches and seller has goods: buyer gets the difference between the market price (or replacement price) and contract price plus incidental and consequential damages minus expenses saved. Buyer breaches and buyer has goods: seller gets the contract price. Buyer breaches and seller has goods: seller gets the difference between the contract price and market price (or resale price) plus incidental damages minus expenses saved. Lost volume seller: The seller gets lost profits plus incidentals. Tip: A seller is a lost volume seller when there is an unlimited amount of the product available. The MBE fact pattern will make this very clear.

Settlement Offers

Settlement offers are inadmissible to prove liability. Admissions made in connection with settlement offers are inadmissible, except to impeach or to show bias. Offers to plead guilty for a criminal offense are also inadmissible.

Impeachment No EE Allowed (no pooping in garden) Spring Root

Specific Acts of Misconduct Truth opinion - Witness is considered to be EE

High Priority Essay: Criminal Law Fruits of the Poisonous Tree - Exclusionary Rule -Start your essay by identifying the Fourth Amendment - applies to states through 4A -General Rule -Standing -reasonable - The Exclusionary Rule -Fruits of the Poisonous Tree Doctrine -Warrant -Exceptions to Warrant Requirement

Standing: a person has standing to raise a Fourth Amendment challenge if he has an expectation of privacy in the thing searched or seized. General rule: The Fourth Amendment applies to searches or seizures conducted by government agents in areas where the complaining individual has a reasonable expectation of privacy. An agent usually needs a warrant. A search must always be "reasonable;" whether or not there is a search warrant or arrest warrant, the arrest or search must not be "unreasonable." The exclusionary rule is used to deter unlawful police conduct. All evidence seized in violation of the Fourth Amendment is inadmissible in a criminal proceeding. It can be used in a grand jury. Fruit: In addition to excluding evidence which was obtained illegally, any additional evidence obtained either directly or indirectly from the illegal search must also be excluded. Exceptions: Evidence that was obtained from an independent source; evidence that would have been discovered eventually regardless of the illegality; and any intervening act of free will of the defendant which purges the tainted evidence. The Fourth Amendment requires a warrant for a lawful search, unless there is a recognized exception. (Probable Cause needed) o Issued by a Neutral and Detached Magistrate; o Supported by Oath or Affirmation; o With Particularity. (names places searched and the items to be seized) Warrantless searches are searches and seizures conducted without court-issued search warrants. In the United States, warrantless searches are restricted under the Fourth Amendment unless there is an exception to the warrant requirement.

CIV PRO KEY Key Principle #4: Be aware of summary judgment under Rule 56, where a case (or part of a case) is decided in favor of the plaintiff or the defendant without a trial.

Start your essay as follows: "Federal Rule of Civil Procedure 56(a) allows a summary judgment motion to be granted only if there is no genuine issue as to any material fact and the movant is entitled to judgment as a matter of law." Note that "a motion for summary judgment may be supported by depositions, documents, electronically stored information, affidavits or declarations, stipulations, admissions, interrogatory answers, or other materials." The moving party must produce evidence to show there is no genuine issue of material fact. The burden then shifts to the nonmoving party, which must then produce evidence to show that there is a genuine issue of material fact for trial. The motion is looked at in the light most favorable to the nonmoving party

Subsequent Remedial Measures

Subsequent remedial measures are inadmissible to prove negligence, culpability, a defect or a need for a warning. But it is admissible to prove ownership, control, or to show that precautionary measures were possible.

Non-Hearsay (Wittness is at Trial) Thoughtless Wampas, Idiot Driods, Impregnated Chewbacca.

TESTIFYING WITNESS 1) Prior statement of identification 2) Prior inconsistent statement (made under oath at a formal trial, hearing, or deposition and the declarant is at trial subject to cross-examination concerning the statement) 3) Prior consistent statement used to rebut a charge of recent fabrication of improper motive/influence

Key Principle #5: Temporary remedies, like preliminary injunctions and temporary restraining orders (TROs), are occasionally tested on the MEE. On the MEE, the party in the fact pattern generally seeks a TRO until the court decides whether to grant a preliminary injunction. So, generally, you should discuss both TROs and preliminary injunctions in your MEE answer.

TROs: A TRO can be issued without notice to the adverse party (but only in limited circumstances and for a limited time). To secure a TRO without notice, the plaintiff needs to show a risk of "immediate and irreparable injury." The TRO lasts only long enough for the court to consider and resolve a request, but not longer than 14 days (unless the court extends it for good cause or the adverse party consents to an extension). TROs are considered to be "stop-gap" measures and last until the court decides whether to grant a preliminary injunction. Preliminary injunctions: A preliminary injunction is equitable relief with the objective of preserving the status quo. If it is granted, the matter must be tried within six months unless the parties stipulate or good cause is shown. The court must give notice to the adverse party. There are four factors to consider (mnemonic = "HELP"): (1) harm: the significance of the threat of irreparable harm to the plaintiff if the injunction is not granted, (2) evaluation of injuries: the balance between this harm and the injury that granting the injunction would inflict on the defendant, (3) likelihood of prevailing: the probability that the plaintiff will succeed on the merits, and (4) the public interest.

A victim and his former business partner, the defendant, had a bitter falling out after the victim accused the defendant of embezzling company funds. The defendant threatened to get even. Shortly thereafter, while driving on the expressway, a car swerved suddenly in front of the victim's car. Although the victim applied the brakes immediately, his car failed to stop. To avoid colliding with the car ahead of him, he swerved to the right and smashed into a concrete retaining wall. A passing motorist stopped and came to the aid of the victim. Bleeding profusely from a head wound, and rapidly losing consciousness, the victim said, "I don't think I'm going to make it. I tried to slow down, but my brakes didn't work. My former partner must have tampered with them to get back at me." With that, the victim lapsed into unconsciousness, and has been in a coma and on life support ever since. A personal injury suit has been filed on his behalf by a court-appointed guardian against the defendant. At trial, can the motorist testify as to the statement made by the victim?

Testimony as to the statement made by the victim is inadmissible as a statement under belief of impending death, because the victim did not actually have firsthand knowledge that the defendant was responsible for the collision. The statement is hearsay because it is a statement made by the declarant (the victim), other than while testifying, offered to prove the truth of the matter asserted therein. Here, the plaintiff wants to present this testimony to prove the truth of the statement that the defendant was responsible for the brake failure, and will argue that the statement falls under the hearsay exception for dying declarations. In a civil case or a homicide prosecution, a statement made by a now unavailable declarant while believing his death to be imminent, that concerns the cause or circumstances of what he believed to be his impending death, is admissible. [Fed. R. Evid. 804(b)(2)] For this exception to apply, the declarant need not actually die. Rather, the declarant must be "unavailable" when the statement is offered. A declarant is unavailable if he: (i) is exempted from testifying on the ground of privilege, (ii) refuses to testify despite a court order, (iii) testifies to lack of memory of the subject matter of the statement, (iv) cannot be present or testify because of death or physical or mental illness, or (v) is beyond the reach of the court's subpoena and the statement's proponent has been unable to procure his attendance or testimony by process or other reasonable means. Regarding the statement at issue here, the victim certainly thought he was about to die from his injuries. In addition, he is unavailable, as his physical condition prevents him from testifying. However, the victim's statement represents a mere suspicion that the defendant tampered with the brakes. As well-founded as such a suspicion may be (given the history between the victim and the defendant), a statement based on mere suspicion rather than actual knowledge does not constitute a statement concerning the cause or circumstances of an "impending death" for purposes of the dying declarations exception. Thus, (A) is the correct answer and (C) is incorrect. (B) is incorrect because the declarant's death is no longer required; unavailability is sufficient. Thus, if the victim's statement otherwise qualified under the dying declarations exception, the fact that he is not dead would not render the motorist's testimony inadmissible. (D) is incorrect for the reasons stated above and also because it incorrectly implies that the dying declarations hearsay exception applies only in civil cases. As noted above, the exception also applies to homicide cases. (Note that the traditional view, still followed by some states, would only allow the declaration in a homicide prosecution.)

An accountant employed by the Federal Communications Commission was offended by various jokes and cartoons that employees would post in the office cafeteria. The Commission did not have any rules regarding what employees could post in the cafeteria, and none of the cartoons were pornographic or harassing. Nevertheless, the accountant lodged a number of complaints with his supervisor that went unheeded. Finally, the accountant posted his own notice chastising the hypocrisy and immorality of the agency for allowing such cartoons when it was charged with ensuring a standard of decency on the public airwaves. The notice prompted a great deal of debate among employees and a great deal of displeasure on the part of the accountant's supervisor, particularly after it was posted on another employee's blog and received some media attention. A labor contract between the agency and the clerical workers' union contained a policy for providing for termination of union employees only for certain specified grounds, but the accountant was not a member of the union and was not covered by the policy or any other employment agreement. Which of the following statements is most accurate regarding the agency's right to dismiss the accountant?

The accountant is entitled to a hearing because he has a liberty interest in the exercise of his First Amendment rights. If the accountant is fired, he has a right to a hearing to determine whether his First Amendment rights were violated by his dismissal. Under the Due Process Clause of the Fifth Amendment, a person has a liberty interest in the exercise of specific rights provided by the Constitution, including freedom of speech. A government employee may not be fired for expressing his views regarding public issues, but can be fired for speech that disrupts the employer's policies or undermines the employer's authority

A state statute makes criminal "all speechmaking, picketing, or public gathering of any sort on the steps of the supreme courthouse Monday through Friday, between the hours of 8:30 a.m. and 4:30 p.m., when court is in session." A citizen is upset about a supreme court decision that was just released and stands on the steps of the courthouse at noon, while court is in session, handing out leaflets and exhorting passersby to vote the state supreme court justices out of office. If the citizen is prosecuted for violation of the statute, which of the following best describes the applicable burden of proof?

The citizen will have to show that there was no reasonable basis for enacting the statute.

Essay Topic: High priority Probative Value/Relevancy - Relevancy Def - 403 Rule -There are other specific exclusions: PRISM

The condition for admitting evidence must be relevant. Relevance includes probative and material evidence. Evidence is relevant if it has any tendency to make a fact more or less probable than it would be without the evidence, and the fact is of consequence in determining the action. 403 Trial judge has discretion - Otherwise, relevant evidence may be excluded if its probative value is substantially outweighed by the danger of: • Unfair prejudice; • Waste of time; • Confusion. • Character-related evidence; • Habit and custom; • Subsequent remedial measures; • Payment of medical and related expenses; • Settlements and plea bargains; • Liability insurance.

To help alleviate discrimination in private contracts, Congress passed a bill providing: "It shall be unlawful to discriminate against minority race members in the making and enforcement of any public or private contract, of every kind whatsoever. Any person whose rights under this statute are violated may bring a cause of action against the party that has so violated the person's rights in the federal district court for the district in which he resides, seeking treble damages or $1,000, whichever is greater." Several large banks that have been accused of discriminatory loan practices challenge the federal statute. If the court finds that Congress had the power to enact the statute, the court most likely will find that the power arose from which of the following?

The court most likely will find that Congress had the power to enact the legislation under the Thirteenth Amendment. The Thirteenth Amendment simply provides that neither slavery nor involuntary servitude shall exist within the United States and gives Congress the power to adopt appropriate legislation to enforce the proscription. Since the amendment is not limited to proscribing state action, Congress may adopt legislation regulating private parties. Under the amendment, the Supreme Court has allowed Congress to prohibit any private conduct that Congress deems to be a "badge" or "incident" of slavery, and has upheld statutes regulating private contracts. [See, e.g., Runyon v. McCrary (1967) . (C)-the Fourteenth Amendment-is incorrect. The Fourteenth Amendment prohibits states from discriminating on the basis of race; it does not extend to private conduct

In an accountant's trial for filing fraudulent tax reports, the prosecution calls a former colleague of the accountant, and she testifies that the accountant's reputation in the community is for frequently participating in very questionable reporting, often resulting in unnecessary risk for his clients. She testifies further that she thinks the accountant is dishonest. Should the trial court admit this evidence over the accountant's objection?

The court should not admit this evidence because the prosecution cannot initiate evidence of the defendant's bad character. The prosecution may offer such evidence only after the accused has put his character in issue by either taking the stand (thus placing his credibility in issue) or offering evidence of his good character. Thus, (C) is incorrect. (B) is incorrect because, under the Federal Rules, character may be proven by opinion evidence. (D) is incorrect because this does not constitute a regular response to a specific set of circumstances; it is merely reputation and opinion evidence.

The criminal statutes of the state define manslaughter and murder as they were defined at common law. As to insanity, the state has the following provision: "Under the defense of insanity a defendant may be entitled to acquittal if, because of mental illness, the defendant was unable to control his or her actions or to conform his or her conduct to the law." The defendant was put on trial in the state for the murder of his wife and her co-worker. The evidence at trial established that the defendant's wife was having an affair with the co-worker, and that the defendant learned of it and killed the pair. The defendant did not take the stand in his own defense. In his closing statement to the jury, the defendant's attorney made a statement, "Ladies and gentlemen, you must consider that there are some things that would provoke any one of us to kill, and there are things that make one unable to control one's actions." The defendant's attorney requested that the judge give the jury instructions on manslaughter and on insanity, and the judge agreed to do so. The judge also issued the following instructions: "INSTRUCTION #6: In order to mitigate an intentional killing to voluntary manslaughter, the burden of proof is on the defendant to establish that adequate provocation existed." "INSTRUCTION #8: Insanity is an affirmative defense and the burden of proof is on the defendant to establish that such insanity existed at the time of the killing." The jury found the defendant guilty of murder, and he appealed. He asserts that the jury instructions violated his rights under the federal Constitution. How should the appeals court rule?

The court should reverse the defendant's conviction because Instruction #6 requires the defendant to disprove one of the elements of murder. Due process requires in criminal cases that the state prove guilt beyond a reasonable doubt.

To help reduce a rising crime rate among teenage boys in a city, a scout leader decided to organize an overnight jamboree to get teens interested in scouting. The scout leader met with the city's parks commissioner and requested a permit to camp at a large city park located on the oceanfront. The parks commissioner told the scout leader that a city ordinance prohibited large organized use of the park during the evening and all overnight camping. The commissioner explained that the city wished to keep the park open for general use during the evening, when most people were off work, and the park was cleaned overnight. The scout leader brought an action in federal district court, seeking to compel the city to allow overnight camping for this one special occasion. If the court determines that the ordinance is valid, what will be the basis for its decision?The court will base its decision on its determination that the ordinance is narrowly tailored to serve an important government interest and does not unreasonably limit alternative channels of communication. While the First Amendment protects the freedoms of speech and assembly, the government may reasonably regulate speech-related conduct in public forums through content-neutral time, place, and manner regulation. To avoid strict scrutiny and be upheld, government regulations on speech and assembly in public forums must be content neutral and narrowly tailored to serve an important government interest, and must leave open alternative channels of communication. Here, the ban on camping overnight in the park, a content-neutral regulation of a public forum, would be evaluated by the court using the standard in choice (B). (A) and (D) are incorrect because the rational relationship test is used for restrictions on free speech rights in nonpublic forums. Here, because the park is a public forum, the more restrictive test stated in (B) is used. (C) is incorrect because it misstates both parts of the standard.

The court will base its decision on its determination that the ordinance is narrowly tailored to serve an important government interest and does not unreasonably limit alternative channels of communication. While the First Amendment protects the freedoms of speech and assembly, the government may reasonably regulate speech-related conduct in public forums through content-neutral time, place, and manner regulation. To avoid strict scrutiny and be upheld, government regulations on speech and assembly in public forums must be content neutral and narrowly tailored to serve an important government interest, and must leave open alternative channels of communication. Here, the ban on camping overnight in the park, a content-neutral regulation of a public forum, would be evaluated by the court using the standard in choice (B). (A) and (D) are incorrect because the rational relationship test is used for restrictions on free speech rights in nonpublic forums. Here, because the park is a public forum, the more restrictive test stated in (B) is used. (C) is incorrect because it misstates both parts of the standard.

A homeowner purchased a riding lawn mower from a lawn mower dealer. During his first use of the mower, the homeowner noticed that the mower was vibrating when he turned, but he was able to finish mowing. A few days later, the homeowner lent the mower to his neighbor. The neighbor was driving the mower back to his yard when he made a turn and a wheel broke off, causing the neighbor to be thrown off the lawn mower and onto the sidewalk. The neighbor was injured. The neighbor brought a negligence action against the dealer for his injuries. At trial, the neighbor presented evidence that the wheel broke because of a manufacturing defect. The dealer presented evidence that the homeowner could have discovered the defect after the mower began vibrating when he used it for the first time. In this action, who is likely to prevail?

The dealer will prevail because there is no evidence that it should have discovered the defect. To prove breach of duty in a products liability action based on negligence, the plaintiff must show (i) negligent conduct by the defendant leading to (ii) the supplying of a defective product by the defendant. However, a dealer who buys from a reputable manufacturer with no reason to anticipate that the product is dangerous need make only a cursory inspection of the goods to avoid liability for manufacturing defects. Here, there is no evidence that the dealer should have known that the wheel was defective; hence the dealer will likely prevail. (A) is incorrect because merely selling the lawn mower with an unreasonably dangerous defect, without knowing or being expected to know of the defect, will not subject the dealer to liability for negligence. The statement in (A) is more appropriate in an action based on strict liability. (B) is incorrect because the use of res ipsa loquitur suggested by that choice would be directed at the manufacturer rather than the dealer, and the dealer is not liable for the manufacturer's negligence. (C) is incorrect because the negligent failure of an intermediary to discover a defect is not a superseding cause. If the dealer were otherwise liable, the negligent failure of the homeowner to discover the defect in the wheel would not cut off the dealer's liability.

Duty of Loyalty

The duty of loyalty comes into play whenever a director has a personal stake in an action to be taken by the board. ❖ Bar Professors' Tip: A duty of loyalty is violated if a self-interested director engages in behavior not in the best interest of the corporation. This does not mean that all self dealing transactions violate the duty. The transaction must be considered fair or have the approval of a majority of disinterested directors or shareholders.

A resident being interviewed live by a television reporter stated that, "The biggest problem in this city is corruption in city government, particularly the mayor." The mayor has now brought an action for defamation against the resident. At trial, the mayor has produced testimony as to his honesty and good character. As part of his defense, the defendant seeks to offer into evidence the fact that the mayor was convicted two years ago of taking a bribe to award a city contract for solid waste disposal. Is the evidence admissible?

The evidence is admissible because the mayor's character is directly in issue. The general rule is that evidence of character to prove the conduct of a person in the litigated event is not admissible in a civil case. However, when a person's character itself is one of the issues in the case, character evidence is not only admissible, but in fact is the best method of proving the issue. Where the plaintiff brings a defamation action for injury to reputation and the defendant pleads as an affirmative defense that his statements were true, the plaintiff's character is directly at issue in the case. Under the Federal Rules, any of the types of evidence (reputation, opinion, or specific acts) may be used to prove character when character is directly in issue. [Fed. R. Evid. 405(b)] Here, the mayor's character is at issue and the resident is offering character evidence to show that his assertion that the mayor is corrupt is a true statement. Thus, (C) is correct and (A) is incorrect. (B) is incorrect because, as stated above, any of the types of evidence can be used to prove character when it is directly in issue. (D) is incorrect because an actual conviction is required for impeachment purposes, but not for the purpose of establishing character-evidence of an arrest or indictment would have been equally admissible.

A homeowner bought "20-pound test" fishing line for hanging potted plants on his porch. "20-pound test" in the fishing industry means that fishing line will not break under an initial stress of up to 20 pounds when a hooked fish tugs against the line, but not that it will support a constant 20-pound weight. Most sportfishers are aware of this technical meaning, but most laypersons are not, and the manufacturer put no warnings or explanations on the package in which the line was sold. The homeowner hung a 15-pound basket from his front porch, directly above an old-fashioned porch swing. A friend visiting the homeowner was sitting on the swing when the line holding the basket broke, causing the plant to fall and strike the guest on the head. In a jurisdiction following the traditional rules for landowners and possessors of land, will the guest prevail against the homeowner in a suit to recover damages for her injuries?

The guest will not prevail in a suit against the homeowner. As a social guest of the homeowner's, the guest is deemed to be a licensee; i.e., one who enters onto land with the owner's permission for her own purpose or business rather than for the owner's benefit. In a jurisdiction following the traditional rules for landowners and possessors of land, the owner owes a licensee the duty to warn of or make safe a dangerous condition known to the owner that creates an unreasonable risk of harm to the licensee and that the licensee is unlikely to discover. The owner has no duty to a licensee to inspect for defects nor to repair known defects. The homeowner, as a person who was not involved with fishing, had no reason to suspect that a fishing line that was "20-pound test" could not support the constant weight of a 15-pound basket. Thus, the homeowner did not know of the dangerous condition present in the form of the basket overhanging his porch. Because the homeowner was unaware of the danger, he was under no duty to warn the guest, a licensee, of the dangerous condition. Having violated no duty owed to the guest, the homeowner will not be held liable for her injuries. (A) is accurate in stating that the guest was a social guest. However, as detailed above, the duty owed to a guest is simply to warn of concealed dangerous conditions of which the owner is aware. The homeowner had no duty to warn of a danger of which he neither knew nor had reason to know. (B) is incorrect because there is no indication either that the homeowner hung the basket in a negligent manner or that he was negligent in failing either to warn the guest or to be aware of the danger. The homeowner appears to have acted as would a reasonable person with no knowledge of the meaning of technical terms of fishing. (D) is incorrect because a social guest would indeed be a foreseeable plaintiff. If the homeowner had been negligent in hanging the basket directly above the swing, it would have been reasonably foreseeable that an injury would befall any person who sat on the swing. Thus, (D) reaches the correct result that the guest will not prevail, but for an incorrect reason.

A landowner had a swimming pool and a dressing cabana constructed in her spacious backyard. The pool was entirely within the confines of the landowner's property. However, one corner of the cabana extended a few inches onto a far corner of her neighbor's land. At the time of the construction, neither the neighbor nor the landowner was aware that the cabana extended onto the neighbor's property. Does the neighbor have a cause of action for trespass?

The neighbor will prevail because the cabana extends onto the neighbor's land. The tort of trespass to land requires: (i) an act of physical invasion of the plaintiff's real property by the defendant, (ii) intent by the defendant to bring about a physical invasion of the property, and (iii) causation. The intent required is the intent to enter on a particular piece of land, rather than intent to trespass. Also, it is not necessary that the defendant personally enter the land. It is sufficient if the defendant's act or something set in motion thereby causes a physical invasion of the property. By having the cabana constructed, the landowner acted so as to bring about the physical invasion of the neighbor's land.

Physician-Patient Privilege

The right of a physician to refuse to testify in a trial or other legal proceeding about any statement made to him/her by a patient, on the basis that any communication between the doctor and the patient is confidential.

Right of Redemption

The right to buy back. A debtor may buy back or redeem his or her mortgaged property when he or she pays everything due on the debt.

A college student borrowed his roommate's notebook computer without permission because he needed to write a term paper that was due the next day. While the computer was sitting open on the student's desk overnight, a water pipe in the ceiling began leaking and water dripped down on the computer, rendering it inoperable. A computer repair service estimated that it would cost $500 to repair all the damaged components. At the time it was damaged, the computer was worth $700. If the roommate sues the student for the damage caused to the computer, what will be the extent of his recovery?

The roommate can recover $700 in damages from the student for conversion. To establish a prima facie case of conversion, the following elements must be proved: (i) an act by defendant interfering with plaintiff's right of possession in the chattel, (ii) intent to perform the act bringing about the interference with plaintiff's right of possession, (iii) causation, and (iv) damages-an interference that is serious enough in nature or consequence to warrant that the defendant pay the full value of the chattel. Even if the conduct is wholly innocent, liability may attach where the interference is serious in nature. Accordingly, accidentally causing damage to another's chattel may constitute a conversion when the damage occurred while the defendant was using the chattel without permission. Here, the student interfered with the roommate's right of possession in the computer by taking it without permission, and it sustained damages of over 70% of its value while in the student's possession. Hence, the student has committed a conversion. The plaintiff in a conversion case is entitled to damages for the fair market value of the chattel at the time and place of the conversion, which in this case was $700. (A) is incorrect because even though the student was not at fault in the water pipe leaking, the damage occurred while the computer was wrongfully in his possession. (B) is incorrect. Had the computer not been damaged, the roommate's recovery would be limited to loss of use damages under a trespass to chattels theory. However, the serious damage that occurred while the computer was in the wrongful possession of the student warrants a recovery for conversion. (C) is incorrect because the damages remedy for conversion is the fair market value; in effect, there is a forced sale of the item. The student may keep the computer but he is liable to the roommate for the entire value of the computer rather than just the cost of repairs.

To combat rising insurance rates, a state formed a state-owned insurance company that operated exclusively within the state. The company provided insurance on the basis of premiums calculated according to a schedule of fees. Under the schedule, premiums for residents of a particular city were 25% higher than the premiums for any other municipality in the state. Forty percent of that city's residents were of Mexican descent compared with a state-wide Mexican-American population of approximately 15%. A Mexican-American citizen living in the city brings suit, alleging that the state insurance company's rate structure violates the Equal Protection Clause. Will the citizen's suit prevail?

The state insurance company will prevail unless the citizen can show that the company charges Mexican-American citizens higher rates than other citizens of that city who are similarly situated. The mere fact that legislation or governmental action has a discriminatory effect is not sufficient to trigger strict scrutiny. There must be intent to discriminate on the part of the government, which can be shown by the discriminatory application of a law or regulation that appears neutral on its face. If the state insurance company is charging the city's Mexican-American citizens higher rates than citizens who are otherwise situated the same, the court will find that there is an intent to discriminate in the rate-setting process, triggering strict scrutiny because a suspect class is involved. (A) is incorrect because, as stated above, the citizens must show more than a discriminatory effect to prevail. The classification will be subject to strict scrutiny only if an intent to discriminate is established, which can be shown by (i) facial discrimination, (ii) discriminatory application, or (iii) discriminatory motive. (B) is incorrect. If the strict scrutiny standard applied, proof of a compelling interest would be required to uphold the discriminatory classification. However, as discussed, strict scrutiny will be triggered only if an intent to discriminate is shown; a discriminatory effect is not sufficient. (D) is incorrect because government actions or regulations that improperly discriminate against a suspect class may violate equal protection even if they are "economic" in nature.

Congress passed a law allowing widespread oil exploration on federal lands in the western United States. A large deposit of oil sand was discovered in one western state and Congress authorized an oil sand refining plant to be built on federal park land within the state. The refinery was built in compliance with federal pollution regulations. Pursuant to state law, the plant manager allowed the state to inspect the plant before putting it into operation. Because state refinery standards were more strict than the federal standards (in order to better protect state citizens from pollution associated with refineries), the refinery did not pass the inspection, and the state inspector refused to give the manager a permit to run the refinery. The refinery manager nevertheless began to run the refinery and was fined by the state.What is the manager's best defense against imposition of the fine?

The state law violates the principles of intergovernmental immunity as applied to the manager.The states have no power to regulate the activities of the federal government unless Congress consents to the regulation. Thus, instrumentalities and agents of the federal government are immune from state regulations that interfere with their federal functions. Here, the regulation clearly interferes with the manager's duties to run the refinery. While it might be argued that the manager agreed to comply with the state regulations, because he allowed the state inspection, nothing indicates that Congress consented, and so the state regulation cannot be applied to the manager.

A swimmer went to a privately owned lake resort whose owner charged a fee for admission. The beach had a roped-in swimming area and large signs directing swimmers not to swim anywhere but within the ropes. The lifeguards regularly enforced this rule. The resort also rented canoes and rowboats to its patrons, who could take them anywhere on the lake. The swimmer and two of his friends had rented a canoe and started to paddle out toward the other side of the lake when the swimmer saw a volleyball game starting on the beach that he wanted to join. He left his friends in the canoe and started swimming to shore. He was only a few yards outside of the roped-in swimming area when he started, but he angled away from the swimming area toward the area of the beach where the volleyball net was set up. Although the lifeguard on duty saw him, she did not warn him to return to the swimming area. When the depth of the water was about four feet, he put his foot down and was severely cut by the jagged edge of a rusted metal stake protruding a few inches out of the bottom of the lake. The swimmer had not seen the stake even though the water was clear and it was visible if he had looked down. If the swimmer sues the resort in a jurisdiction that applies the traditional rules for landowners and possessors of land, is he likely to recover?

The swimmer cannot recover from the resort because he did not have invitee status when he was injured. In jurisdictions following the traditional rules for landowners and possessors of land, the nature of a duty of an owner or occupier of land to those on the premises depends on the legal status of the plaintiff in regard to the property, i.e., whether the plaintiff is a trespasser, licensee, or invitee. An invitee is a person who enters onto the premises in response to an express or implied invitation of the landowner. Those who enter as members of the public for a purpose for which the land is held open to the public and those who enter for a purpose connected with the business or other interests of the landowner are considered invitees. However, a person will lose his status as an invitee if he exceeds the scope of the invitation-if he goes onto a portion of the property where his invitation cannot reasonably be said to extend. Here, the swimmer was an invitee of the resort in the areas to which it allowed its patrons to go. However, the resort clearly identified the boundaries of the area held open to swimmers, and the swimmer could not reasonably have believed that he was invited to swim in the area where he was injured. Because the swimmer was at most a licensee when he was injured, the resort did not owe him a duty to make reasonable inspections of that area to discover dangerous conditions and make them safe. At most, the resort had a duty only to warn the swimmer of known dangerous conditions that create an unreasonable risk of harm to him and that he is unlikely to discover, and nothing in the facts indicates that any employees of the resort knew of the stake under the water. The swimmer therefore cannot recover against the resort. (A) is not as good a choice as (B). While a landowner is not liable for a dangerous condition that is obvious to the entrant on the land, the fact that the stake was visible does not establish that it was obvious, given that the swimmer was looking forward rather than down. Whether a danger is obvious is determined by all of the surrounding circumstances, not just whether the danger is visible. The better reason why the swimmer cannot recover is because he was no longer an invitee. (C) is incorrect because the lifeguard's failure to direct the swimmer to the swimming area would not constitute an invitation to swim in the restricted area; at most, it would establish only that the swimmer was a licensee rather than a trespasser when he swam in that area. A licensee is one who enters onto land with the possessor's permission, express or implied, for his own purpose or business rather than for the possessor's benefit. The lifeguard's conduct may have constituted implied permission for the swimmer to exit the lake in a nonswimming area for his own benefit, but it does not establish that he reasonably believed that he was invited to swim in that area. (D) is incorrect because the swimmer lost his status as an invitee when he exceeded the scope of his invitation by swimming in an area where swimming was not permitted.

In the course of repainting an apartment, the landlord of a small apartment building used a professional strength, stain-killing primer manufactured by a paint company for professional painters. The building's common ventilation system was running as the landlord applied the primer, and some fumes from the primer went through the ventilation system into the apartment of the upstairs tenant, who suffered injuries to her eyes as a result. The warning label on the can, which the landlord read, stated: "Danger. This material is extremely hazardous and volatile. Do not use near open flame. Use only with adequate ventilation." The product contained a chemical known to be harmful to people's eyes, but in the 15 years that the product has been on the market, there were no reported cases of anyone suffering an eye injury from the product. However, professional painters routinely close off or shut down any common ventilation systems in buildings before using the product. If the tenant brings an action against the paint company on a theory of strict liability, will she recover?

The tenant will likely prevail because the lack of a warning about eye injuries made the product unreasonably dangerous. A products liability action based on strict liability requires the following: (i) the defendant is a commercial supplier; (ii) the defendant produced or sold a product that was defective when it left the defendant's control; (iii) the defective product was the actual and proximate cause of the plaintiff's injury; and (iv) the plaintiff suffered damage to person or property. Here, the paint company is a commercial supplier of a "defective" product. Although the primer was not actually defective in that it apparently performed as it was meant to do, it is legally defective if it was unreasonably dangerous and could be made safer by adequate warnings. Here, the paint company knew of the danger and could easily have placed a specific warning on the label. Even though professional users may have known of the danger, it was not obvious, and it could have been avoided at minimal cost by including a specific warning. That would have alerted the landlord to the danger, making it more likely that he would take precautions that would have prevented the tenant from being injured. To prove actual cause where the plaintiff's claim is that the product is defective because of lack of an adequate warning, the plaintiff is entitled to a presumption that an adequate warning would have been read and heeded. Thus, the tenant can likely establish liability on her cause of action.

A defendant was charged with the murder of a victim. During the course of the criminal trial, a witness testified on behalf of the defense that, at the time the murder took place, he saw someone who looked like the defendant dancing at a local nightclub. The defendant is eventually acquitted of the charge. Following the acquittal, the appropriate survivors of the victim bring a wrongful death action against the defendant. As part of her defense, the defendant wishes to introduce the testimony given at the criminal trial by the witness, who the defendant shows is now incarcerated in a prison in another state.The testimony of the witness is:

The witness's testimony is inadmissible.Under Federal Rule 804(b)(1), the testimony of a witness who is unavailable, given at another hearing, is admissible in a subsequent trial if there is sufficient similarity of parties and issues so that the opportunity to develop testimony or cross-examination at the prior hearing was meaningful.The former testimony is admissible upon any trial of the same subject matter. The party against whom the testimony is offered or, in civil cases, the party's predecessor in interest must have been a party in the former action. "Predecessor in interest" includes one in a privity relationship with the party, such as grantor-grantee, testator-executor, life tenant-remainderman, and joint tenants.These requirements are intended to ensure that the party against whom the testimony is offered (or a predecessor in interest in a civil case) had an adequate opportunity and motive to cross-examine the witness.In the civil suit here at issue, the survivors of the victim were not parties to the criminal case, nor were they in privity with any such party. (The parties to that case were the defendant and the government.) These survivors, who are the plaintiffs in the instant litigation, are the parties against whom the testimony of the witness is being offered. Because they were not parties to the action in which the witness testified, they had no opportunity to cross-examine him. Even if the government had a similar motive to cross-examine the witness as do the plaintiffs in the current action, that is not sufficient to make the government a predecessor in interest to the plaintiffs. Consequently, the testimony of the witness does not come within the former testimony exception to the hearsay rule, and the testimony is inadmissible hearsay.

A man boarded a plane with his pet rattlesnake hidden in his carry-on bag. The man was not aware that the carry-on bag, which he put under the seat in front of him, had a defective zipper. The snake, which had no teeth or venom and was harmless, escaped from the bag and started slithering down the aisle while the man was using the lavatory. A woman who had just gotten up from her seat saw the snake heading towards her and tried to run in the other direction. She tripped over someone's foot and broke her ankle. If the woman sues the man on a theory of strict liability for her broken ankle, will she prevail?

The woman will prevail because the rattlesnake is classified as a wild animal. An owner of a wild (i.e., nondomestic) animal, even one kept as a pet, will be strictly liable for the damage caused by the animal. A rattlesnake, even a harmless one, will be classified as a wild animal. Therefore (C) is correct and (A) is wrong. (B) is wrong because the injury the woman suffered was within the "normal dangerous propensity" of the animal. Strict liability for wild animals includes liability for the harm that results when a person is attempting to flee from what is perceived to be a dangerous animal. (D) is wrong. The fact that the activity was uncommon in the locale would have some relevance if the lawsuit were based on a theory of strict liability for an abnormally dangerous activity. It has nothing to do with strict liability for damage caused by animals.

Tere are two separate privileges related to marriage. There is spousal immunity, under which: There is also a privilege for confidential marital communications,

There is spousal immunity, under which: (i) a married person whose spouse is the defendant in a criminal case may not be called as a witness by the prosecution, and (ii) a married person may not be compelled to testify against her spouse in any criminal proceeding. In federal court, this privilege belongs to the witness-spouse so that she may not be compelled to testify, but neither may she be foreclosed from testifying. This privilege terminates upon divorce. There is also a privilege for confidential marital communications, under which either spouse, whether or not a party, has a privilege to refuse to disclose, and to prevent another from disclosing, a confidential communication made between the spouses while they were married. Both spouses jointly hold this privilege. Divorce does not terminate this privilege retroactively. Because the communication must be made in reliance upon the intimacy of the marital relationship, if the communication is made in the known presence of a stranger, it is not privileged. Similarly, if one spouse voluntarily reveals the contents of the communication to a stranger, that spouse waives the protection of the privilege as to herself (i.e., she cannot use the privilege to refuse to disclose, or to prevent another from disclosing, the communication), but the other spouse (i.e., the one who did not reveal the communication) retains this privilege.

Due Process Substanative state action interferes with fundamental right Public Speech

Time Place Manner - public forum or deg. public forum content neutural and narrowly tailored

A state provided for a public school system based primarily on property tax revenues from the various districts. School districts that had a property tax base below a certain threshold received supplemental funds from the state that were derived from state lottery revenues. The school districts receiving the supplemental funds served a predominantly Hispanic population as compared to the school districts funded only from property tax revenues. To help balance its budget this year, the state legislature passed a statute terminating the supplemental funds program and earmarking the lottery revenues for deficit reduction. A group of parents of Hispanic schoolchildren in one of the school districts formerly receiving supplemental funds filed suit in federal court, alleging that the state's action in terminating the funding violates the Equal Protection Clause of the Fourteenth Amendment. Which of the following best describes the appropriate standard by which the court should review the constitutionality of the state action?

To prevail, the parents will have to show that the statute does not meet the rational basis test. Under that test, a law is presumed to be valid and will be upheld unless the challenger can make the difficult showing that it is not rationally related to a legitimate state interest. Here, the statute terminating the funds did not target a suspect classification and did not burden a fundamental right, so the rational basis test applies. (A) is incorrect because it is not enough to show that legislation has a discriminatory effect on a suspect classification; there must be an intent to discriminate. To establish a racial, national origin, or ethnicity classification, the party challenging the law must show that (i) the racial classification appears in the law itself (facial discrimination), (ii) the law was applied in a purposefully discriminatory manner, or (iii) the law was enacted or maintained for a discriminatory purpose. None of these situations appears to be indicated under these facts.

Key Principle #1: Regarding validity of a trust, most MEE answers state: "a trust of personal property is valid if it has a trustee, a beneficiary and trust property." A trust is a fiduciary relationship in which a trustee holds legal title to specific property under a fiduciary duty to manage, invest, safeguard and administer the trust assets and income for the benefit of designated beneficiaries, who hold equitable title. There must be a proper purpose for the trust. The trust must be legal and not contrary to public policy.

Trustee: A trustee manages the trust property and holds it for the benefit of the beneficiaries. Note that a trust will not fail for lack of a specifically-appointed trustee. (The court will appoint one.) Beneficiary: In a private express trust, beneficiaries must be definite and ascertainable. Further, the same person cannot be the sole trustee and sole beneficiary or the trust collapses. Trust property (trust res): trust property must be identifiable.

Lay Testimony

Under the Federal Rules, lay opinion testimony is admissible when: (i) it is rationally based on the perception of the witness; (ii) it is helpful to a clear understanding of her testimony or to the determination of a fact in issue; and (iii) it is not based on scientific, technical, or other specialized knowledge.

competence to stand trial

Understanding the charges against one and being able to assist in one's own defense study insantiy

Key Principle #3: When venue is tested, the MEE tends to test general rules. It is important, however, to be well aware of "transfer to a more appropriate forum" since this issue is tested regularly.

Venue is proper in a district where (1) any defendant resides if all defendants reside in the same state, (2) in a district where a substantial part of the events or omissions giving rise to the claim occurred, or (3) a substantial part of property that is subject to the action is situated. (There are also narrow fallback rules which, so far, have not been emphasized on the MEE.) A corporate defendant is deemed to "reside" in any judicial district in which it is subject to personal jurisdiction at the time the action is commenced. To figure out where a corporate defendant resides, divide the state into districts (if it has more than one district) and see if the defendant would be subject to personal jurisdiction in any of those districts. Transfer to a more appropriate forum: Under Title 28, U.S.C. §1404 (2011), the federal court has authority to transfer a case to another federal district for the convenience of the parties and witnesses and in the interest of justice. The new forum must have subject matter jurisdiction and personal jurisdiction. The court will apply the law of the transferor forum. A motion to transfer to a more appropriate forum should be denied if the case could not have been filed there to begin with. (Note: This is different than "transfer to a proper venue" where a case is filed in the wrong venue, and, if transferred, the law of the transferee court would apply. Transfer to a proper venue has not been tested on the MEE.)

Key Principle #5: Unless the court has jurisdiction over the defendant spouse, it may not issue a binding order affecting personal rights such as property division or support, but it may grant a divorce.

Which court may dissolve a marriage: The court does not need jurisdiction over both spouses to terminate a marriage. If the plaintiff spouse is domiciled in the forum state or if the state has some other equivalent long-term connection between at least one of the parties to the marriage, then that court has jurisdiction to dissolve the plaintiff's marriage. Which court may issue a property division order: unless the court has jurisdiction over the defendant spouse, it may not issue a binding property division or support order. Which court determines child custody and support: if a parent wants to acquire jurisdiction over a nonresident parent for child support or custody, then she most show that the state has jurisdiction over the nonresident parent (e.g., minimum contacts). Child support: The Uniform Interstate Family Support Act (UIFSA) governs child support. This Act has been adopted by all states. Once an order is registered, it may be enforced by any state. The state that originally issued a child support order has continuing exclusive jurisdiction to modify that order if the state remains the residence of the obligee, the child, or the obligor, and at least one of the parties does not consent to the jurisdiction of another forum. Child custody order: The Uniform Child Custody Jurisdiction and Enforcement Act (UCCJEA) applies. This is divided into a few tests: Thehomestatetest: The"home state" has exclusive jurisdiction to modify a custody decree. A "home state" is a state where the child has lived with a parent or a person acting as a parent for at least six consecutive months immediately before the commencement of the child custody proceeding. A home state continues to have exclusive jurisdiction to issue a custody order for six months after a child leaves the state, so long as a parent, or person acting as a parent, still lives in the home state. The "significant connections" test: if a child has no "home state," a state may exercise jurisdiction based on (1) "significant connections" with the child and at least one parent and (2) the existence of "substantial evidence" relating to child custody in the forum jurisdiction. Emergency jurisdiction or default jurisdiction: if no other state has or exercises jurisdiction, this test applies.

Lay Witness Competency

Witnesses are presumed to be competent if they have personal knowledge, take an oath to testify truthfully, and are sworn in. B. Personal Knowledge A witness may testify to a matter only if evidence is introduced sufficient to support a finding that the witness has personal knowledge of the matter. Evidence to prove personal knowledge may consist of the witness's own testimony. C. The Dead Man's Statute A dead man's statute states that a party could not testify about personal communications or specific transactions against the estate of a dead person if he has an interest in the outcome of the litigation. Exceptions: testifying as to what happened after the death of the deceased and whether the estate opens the door to the testimony. • Defective Capacity A witness may be impeached by illustrating the witness' lack of perception, intoxication, good eyesight, hearing, loss of memory, etc. • Character Evidence o Reputation and Opinion A party could, using character traits for untruthfulness, impeach a witness using reputation or opinion evidence. o Bad Acts A party could impeach a witness using prior unconvicted bad acts relating to the truthfulness of the witness. The inquiry must be based on good faith questioning. If the witness denies any of the untruthful bad acts, the inquiry stops completely and no other witness can testify to contradict the witness' denial. No extrinsic evidence is allowed. ❖ Bar Professors' Tip: A court may, in its discretion, admit evidence relating to a prior bad act if it is offered during cross-examination of the witness being impeached and is probable of untruthfulness, e.g., giving a false name, false occupation or a false excuse for being absent from work. o Convictions ▪ Felonies A witness can be impeached using past convictions if the conviction is less than ten (10) years old and the court determines that its probative value is outweighed by its prejudicial effect. ❖ Bar Professors' Tip: In a criminal case, the prosecution may impeach the defendant witness with a felony conviction, not involving dishonesty, only if the trial judge determines its probative value outweighs the danger of unfair prejudice. ▪ Untruthfulness A witness may be impeached using conviction of crimes involving dishonesty and false statement, no matter if it is a felony or misdemeanor. The conviction must be less than ten (10) years old. The judge has no discretion to exclude such evidence for prejudicial effect. ❖ Bar Professors' Tip: Character evidence is generally inadmissible to prove action in conformity with the character trait. However, when a person testifies as a witness, that person's credibility becomes a material issue; a witness' credibility thus may be attacked through impeachment by showing the witness has an untruthful character. ❖ Bar Professors' Tip: The Federal Rules expressly prohibits the use of extrinsic evidence to impeach a witness' character for truthfulness. Specific instances of the conduct of a witness, for the purpose of attacking or supporting the witness character for truthfulness may not be proved by extrinsic evidence. Thus, although a witness may be 20 Bar Professors UBE Essay Blueprint cross-examined about a prior alleged lie, if he refuses to admit to lying, he may not be contradicted with extrinsic evidence. F. Rehabilitation of the Witness When the credibility of the witness has been weakened by impeachment, the attorney can rehabilitate the witness by showing a witness' good character or by introducing prior consistent statements, if necessary. G. Direct and Cross Examination Direct examination is the first questioning of a witness in a trial. Leading questions are not permitted on direct examination. Leading questions are those questions that suggest to the witness the questioner's desired answer. Leading questions are ordinarily limited to • Cross Examination • Direct Examination, if it is a o Hostile Witness; o Adverse Party; or o Witness Identified with the Adverse Party. The scope of cross examination is generally limited to the scope of direct examination and the credibility of witnesses. ❖ Bar Professors' Tip: Redirect examination is generally limited to those aspects of the witness' testimony that was first brought out during cross examination, and similarly, re cross is generally limited to matters newly brought up on redirect examination. H. Present Recollection Refreshed A witness' memory can be enhanced by showing the witness a document that describes the relevant event. The document is not admitted into evidence, it is just a way to jog the witness' memory. The document must be related to the subject matter and shown to the adverse party. The adverse party can cross examine the witness on the writing. ❖ Bar Professors' Tip: When an otherwise inadmissible document is shown to a witness to refresh his recollection, the witness must read it to himself. It is improper to allow such a document to be read aloud to the jury, and it may be admitted as an exhibit only if offered by the lawyer who has not used the exhibit to refresh the recollection of the witness. I. Past Recollection Recorded Past recollection recorded is considered an exception to the hearsay rule. If a witness' memory cannot be revived, a party may introduce a memo that the witness made at or near the time of the event. The writing is not admissible - it must be read to the jury.

Key Principle #6: Work product has been tested a few times on the MEE. Know its definition and why it matters if statements constitute work product.

Work product is any material prepared in anticipation of litigation. (If it is not prepared in anticipation of litigation, it is not work product!) Written statements given by witnesses might be discoverable even if they are considered work product if the other party can show substantial need and undue hardship. However, an attorney's mental impressions are never discoverable.

In answer to a radio advertisement, a teenager two months shy of his 18th birthday contracted to buy a late model car from a car dealership. The agreement required a $1,500 down payment with the remainder of the $7,200 price to be paid in monthly installments to a local finance company. The teenager's first eight payments were made regularly until his driver's license was suspended. He then informed the company that no further payments would be forthcoming. The finance company sued for the remaining payments. The age of majority in the teenager's state is 18 years. Would the teenager be liable for the balance of the payments?

Yes, because he kept the car for six months after reaching the age of majority.

A well-known actor sued a resort hotel for damages to his new limited edition sports car caused by the hotel's parking valet while the actor was a guest at the hotel. His lawsuit, based on theories of respondeat superior and negligent hiring, alleged that after he gave the valet the keys, the valet, who had been working for the hotel for nine months, took the car for a drive without permission and negligently drove it into a tree, causing extensive damage to the car. At trial, the actor's counsel offers evidence that six months before the accident, but three months after the valet was hired, the hotel instituted new hiring procedures for all potential employees, including parking valets. Included in the new rules was a requirement that all persons must pass a thorough background check before being hired. The valet had been required only to have a valid driver's license when he was hired. In fact, he had an extensive record of traffic offenses at the time he was hired. Is the evidence regarding the new employment requirements admissible?

Yes, because it is evidence of the hotel's negligence.

The defendant was on trial for driving while intoxicated and injuring a pedestrian. The pedestrian claimed that the defendant was driving the car; however, the defendant's spouse testified at trial that she had been driving the car at the time of the accident and had not consumed any alcohol that evening. In response, the prosecution calls a friend of the defendant's spouse to testify that the spouse told the friend that she and the defendant had changed seats in the car after the incident and that she remained in the driver's seat until the police arrived. Is the testimony admissible?

Yes, for impeachment purposes only.

Key Principle #4: When strict products liability is tested....

You are expected to know that the defendant must be a merchant (rather than a casual seller of goods). Further, under a manufacturing defect theory, the product must be defective from the time it left the manufacturer's hands. All commercial sellers (including a store that sold the product) are liable for manufacturing defects.

unjust enrichment is

a quasi-contract alternative that can be utilized if there is no contract remedy.

Recorded recollection: (hearsay exception)

a witness has insufficient recollection of the event, but he had personal knowledge of the event at a former time, made or adopted a statement while the event was fresh in his memory, and can vouch for the accuracy of the statement when made or adopted.

Cause proximate actual

but for - actual proximate - foreseeable

General Personal Jurisdiction

consent, presence or domicle fed courst may excersize pj to the same extent as court of general jurisdiction in state where dc sits

FSSCS conditional right of entry

estate ends when O enters

Custom - Torts

evidence of duty of care is conclusive in med mal

General rule for damages k

expectation damaages the loss of value and indicentaldamafes plus consequencial damages minus expenses saved as result of breech and mitigation

SMJ - Fed courts have limited Jurisdiction - they can only hear certain cases

fed q diversity Supplemental Juris. juris on one claim but not another - CNNOF.

FSSEI

goes to a third party

An entry of default may be set aside for

good cause shown

Summary Judgement

granted only if there is no genuine issue to any matieral fact and movant is entitled to judgement as a matter of law. burden is on moving party

Refreshing recollection: (presentation of evidence)

if a witness's memory fails him, he may be shown a document or record to jog his memory.

When Breach is an issue - it goes to trier of fact P must show that D breached duty of care - RES IPSA

if p can show res ipsa - there should be directed verdict entered for D

Joint and Several Liability

liability that a person or business either shares with other tortfeasors or bears individually

eviction by ll by 3rd party

ll - no need to pay for entire rent 3rd party - rent is apportioned

FSD - POR

measured by time ends automatically O has POR

Validity of Statues question: is it Fed or State? If it's state...

must meet three part test 1)state and police powers 2) not violate con rights - due process and equal protection 3) must not violate interstate commerce

Due Process Substanative state action interferes with fundamental right unprotected speech

needs to pass rational basis

Easement Creation PING

prescription (AP) Implied - prior use of a ll who subdivides land. Nesescity - land is landlocked G - grant - signed writing

Venue

proper in district where any d resides substantial part of the events or omissions given rise to claim occured or property is located

Warranty deed pReSNt Convents and future

right to convey, seisien, no encumbrances Futher assurances, quiet enjoyment, and warranty

Due Process Substanative state action interferes with fundamental right symbolic speech

ss and narrow tailored to important gov interest

Specific PJ specific actins in state

state courts of general jurisdiction may excersize pj over non-resident defendants who established minimum contacts in the state as to not offend traditional notions of fair play and justice.

Adverse Posession transfer by Will

the gift (adeems)

Joint Tenancy with ROS TTIP Partioned or GASM

time, title, interest, posession giving away, signing k of sale, actual judicial sale, mortgage

Why is it being offered? To prove it was said (nonhearsay)

verbal acts or legally operative words State of Mind Effect on listener / reader (to prove motive or intent)

Key Principle #4: Be familiar with the duty of loyalty and duty of care. The duty of care is heavily tested. TRUSTS

• Duty of loyalty: a trustee has a duty of loyalty to act in the best interest of the beneficiaries. Duty of care—prudent administration: diversify etc. Remedies for a breach of trust: The remedies include: suspending or removing a trustee, decreasing compensation, compelling a trustee to perform trust duties, compelling payment of damages, etc. (There are several other remedies, including asking the court to "order any other appropriate relief.") In a self-dealing case, the trust beneficiaries may rescind the transaction and ask for the self- dealing purchase to be set aside (the trust property is returned to the trust and the amount paid is refunded by the trust) or recover any profits the trustee made by reason of the breach.

Criminal Cases: A defendant in a criminal case could use evidence of character in the following ways:

• Entrapment/First Aggressor Evidence of defendant's character is not admissible in the prosecution's case-in-chief, except if the defendant raises the defense of entrapment or the prosecutor wants to prove the defendant was the first aggressor in a homicide case. • Defendant Opens the Door If the defendant offers evidence of his good character by reputation or opinion evidence, the prosecution may rebut. The introduced character trait has to relate to the charge. Rebuttal is limited to reputation and opinion evidence but the prosecution could ask questions of specific acts about the defendant. • Bad Character of the Victim The defense may use reputation, opinion or specific acts to prove the bad character of the victim and the prosecution may so rebut. The prosecution could offer evidence of the victim's good character as well as the defendant's character for violence. • Sexual Assault Cases Reputation and opinion evidence are inadmissible against the defendant in sexual assault cases but specific acts of sexual behavior by the victim are admissible in the following circumstances: o To protect the defendant's constitutional rights; o To prove the victim consented; o To prove that a person, other than the defendant, is the source of the semen, injuries or other physical evidence of the sexual assault.

Evidence concerning the same time, event or person other than the evidence involved in the instant case is inadmissible with some exceptions:

• Evidence of Ancient History: To prove a fraudulent scheme or to prove damages were caused by a previous accident. • Evidence of a Similar Accident: If the accident is caused by the same instrumentality or condition and occurred under the same or substantially similar circumstances and to prove prior notice to the defendant. It can also be used in reverse - to prove no notice. • Experiments: Under similar circumstances. • Comparable Sales: To prove similar sales if there is a substantial similarity of circumstances. • Industry Custom: To show the appropriate standard of care. • Habit: Admissible to show how the person acted on the occasion at issue. There must be both frequency of the conduct and specificity of the conduct.

Essay Criminal Priority: Receiving Stolen Property Burglary

• Receiving Stolen Property The receiving of property that is known to be stolen with the intent to deprive the owner of it forever. Burglary Burglary requires breaking and entering another's dwelling house, at night, with the intent to commit a felony therein. Thus, once the criminal actor has broken and entered with the appropriate intent, the crime is complete. Even if he has a change of heart once he is inside, he is still liable. ❖ Bar Professors' Tip: The key element is the intent to commit a felony therein. If the defendant intents to go in and play Scrabble, there is no burglary. If he mistakes the house for his own, goes in, and removes jewelry, believing it is his own, there is no intent to commit a felony, and thus, no burglary. ❖ Bar Professors' Tip: Keep in mind that, where intent is concerned, even an unreasonable belief can be a defense, if it negates intent. In real life, a defendant's unreasonable belief may make it far less likely a jury will believe him, but this does not change the legal rule that even an unreasonable belief can negate intent.

expert testimony

• Specialized knowledge helps the jury understand evidence or determine a factual issue; • The witness is qualified as an expert; • The testimony is factually based; • The testimony is based on reliable theories ("good" vs. "junk" science); and • The witness reliably applied the theories to the facts. An expert can base her testimony on: • Facts or data obtained from either o First-hand knowledge; o Observation of other witnesses and evidence; or o A hypothetical posed by counsel. An opinion can be based on otherwise inadmissible evidence if it is of the type reasonably relied upon by experts in the field. But it cannot be disclosed to the jury unless: • The court finds the helpfulness to the jury substantially outweighs the prejudicial effect; or • The adversary inquiries into it on cross-examination. An expert is not required to explain the basis of his opinion on direct but can be asked about the underlying facts or data on cross examination. ❖ Bar Professors' Tip: Expert testimony will be most appropriate when it involves the interpretation of facts of a sort that lay persons are not usually called upon to evaluate.


Kaugnay na mga set ng pag-aaral

BIOL110 Exam 3 Mastering Bio Questions

View Set

(2.1) Elasticity Cross price, income, and supply

View Set

Renal/Immune Practice Q’s - Alicia study sesh 10/23

View Set

[Life/Health License] - CH 6 - Group Life Insurance

View Set